You are on page 1of 196

TOEIC FOCUS ---- TOEIC BUILDING-----Smart Home for Toeic

TOEIC LISTENING
PART 1: PICTURE DESCRIPTION
LESSON 1: VOCABULARY
TOPIC 1: Từ vựng lên quan đến nhà hàng, mua sắm, thời gian rảnh rỗi.
Mua sắm:
- souvenir - be hanging from the rack
- be arranged - be laid out for sale
- be stacked up - be marked down
- be lined up - be stocked with
- be on display - cash register
- browse in the store - reach for an item
- compare prices - piles of bags
- make a selection from - on the shelves
- on both sides of the aisle - push a shopping cart

Examples:

 Many items have been arranged on the display


table.
 Souvenirs are being sold at an outdoor market.
 Some merchandise is on display.

Nhà hàng:
- chef - be occupied
- stacks of plates - help oneself to refreshments
- dine out - be crowded with patrons
- leave a tip - remove loaves of bread
- pour water into a glass - be seated on the stool
- call the waiter over - take an order

1
- study the menu - sit on the patio
- be covered by a cloth - wait tables
- cluttered table

Examples:

 There are several glasses on the table.


 The dining room has been the repaired.
 Folded napkins are placed on table.

Thời gian rảnh rỗi:

- wave in the breeze - water the plants


- rest outdoors - be seating with one‟s legs crossed
- extend over the hill - spray water into the air
- upside-down - grassy area
- be positioned - hold a fishing pole
- walk toward an archway - row a boat
- admire sculptures - in the opposite direction
- hilltop - be reflected in the water
- be seated in a row

Examples:

 The path goes between the rocks.


 A woman is riding her bike in a mountain area.
 The area is covered with rocks and sand.

 Bƣớc 1: Tích lũy từ vựng cơ bản.


pull out of the water be piled up
porch doorway
be placed be on display on the rack
take a food order serve food
be loaded with cargo in front of
near wallpaper
kitchen utensils look into the store
decide what to eat an outdoor shop
try on a pair of tie up a boat
shoes pay for the on both sides
products lead up to be stocked with items
headboard be sorted into boxes
centerpiece be displayed for sale
browse the shelves do grocery shopping
be laid out chop vegetables

 Bƣớc 2: Nghe từ vựng sau đó điền vào chỗ trống.


1. The sandals on the ground.
2. The hats .
3. A woman is .
4. The boat has been .
5. The boat is .
6. The table for a party of five.
7. They‟re stopped .
8. A man is at the dock.
9. There are lamps of the bed.
10. There are the front door.

 Bƣớc 3: Nghe câu và chọn đáp án đúng.


1. A. The picture is .
B. There is some .
C. The painting has been hung .
D. There is .
E. The shelves are .

2. A. The shelves .
B. The items .
C. The clerk is with products.
D. There in the store.
E. All the bottles .

3 A. The boat is .
B. The boat is .
C. The waves are .
D. in the lake.
E. The boat is .

4 A. People are the stairs.


B. The woman is .
C. The woman is .
D. The woman is .
E. The staircase is the next level.
5 A. The man is .
B. The man is .
C. The man is .
D. Vegetables are .
E. The man is .

6 A. Water is .
B. People are .
C. People are .
D. People are .
E. People are .

TOPIC 2: Từ vựng liên quan đến giao thông, công


trƣờng. Giao thông:
- float in the water - lamppost
- pedestrian - train track
- be packed with - be towed away
- be parked on the ground - be unoccupied
- be lined up in the rows - be tied up in the harbor
- block the road - board the vehicle
- on the deck of the boat - near the platform
- in the back of the truck - dock
- put gas in the car - curb

Examples:

 People are getting on board.


 The bus has stopped to let people on.
 The front door of the vehicle is open.
TOEIC FOCUS ---- TOEIC BUILDING-----Smart Home for Toeic

Công trƣờng:
- Path - railing
- sweep - stairway
- climb a ladder - construction site
- be renovated - hold a shovel
- kneel in the garden - be mounted on the wall
- push the wheelbarrow - dig in the ground
- operate heavy machine - wear a safety hat
- vacuum the floor - pave the road
- lead to - brick
- change a light bulb - load a box
- fence
Examples:

 The worker is reviewing some plans


 There is a blueprint spread out on the beach.
 The man is leaning on the counter.

 Bƣớc 1: Tích lũy vốn từ vựng cơ bản.


walk down the street operate heavy machinery
driveway (n) inspect the car
motorist (n) pull into
resurface the road wait at the taxi stand
repair the lamppost be under construction
push the wheelbarrow change a flat tire
wait at the curb be lined up
intersection (n) into the distance
runway (n) climb the ladder

6
use a hammer board the train
get out of the taxi final destination
sweep the road be on the crosswalk
lay a concrete side walk wait to cross
overpass (n) be under renovation

 Bƣớc 2: Nghe từ vựng, sau đó nghe câu.


1. The pedestrians are .
2. The train is the station.
3. Some people .
4. A motorcycle is .
5. The building is .
6. The road curves .
7. The man is .
8. People are the street.
 Bƣớc 3: Nghe và chọn đáp án đúng.
1. A. He is .
B. He is .
C. The bus .
D. The bus is .
E. He is .

2. A. The umbrellas are .


B. The outdoor café is .
C. Most chairs are .
D. People are on the sidewalk.
E. Some people are .
TOEIC FOCUS ---- TOEIC BUILDING-----Smart Home for Toeic

3. A. The man has .


B. The man is at the moment.
C. The man is .
D. The man is .
E. The man is by himself.

4. A. None of the people is .


B. The workers are .
C. The workers are to
construct a building.
D. The building is .
E. The construction is .

5. A. Some trees are .


B. The cars are .
C. The sky is .
D. The road is .
E. The traffic is .

6. A. The band is .
B. The trees are .
C. along the street.
D. The lot is .
E. The road .

TOPIC 3: Từ vựng liên quan đến nơi làm việc


Nơi làm việc
- applaud the speaker - doorway

8
TOEIC FOCUS ---- TOEIC BUILDING-----Smart Home for Toeic

- folder - look through a report


- look into a microscope - stare at a screen
- machine lid - gesture at the board
- speak into a microphone - be seated at a workstation
- make photocopies - sort through paperwork
- adjust the equipment - reach for a knob
- wear protective glasses - hold the receiver to the ear

Examples:

 There are several people on the stage.


 The audiences are listening to a speaker.
 Some of the seats are unoccupied.

 Bƣớc 1: Tích lũy vốn từ vựng cơ bản.


copier (n) make a phone call
unoccupied desk (n) workstation (n)
do the paperwork distribute the papers
file cabinet (n) enter the office
drawing table (n) hang up the phone
hand the paper phone someone
view some documents look into the drawer
read through pages read from the screen
address (v) office supplies (n)
face the screen directly talk in a discussion
reach into the cabinet attend the conference
check information deliver the speech
turn the newspaper fix the fax machine

9
type on the keyboard share the office space
conduct a conversation organize the paper

 Bƣớc 2: Nghe từ vựng, sau đó nghe câu.


1. She is on the monitor.
2. He is .
3. The speaker is the audience.
4. The man is .
5. She is directly.
6. The woman is to the man.
7. She is on the stage.
8. He is at the desk.
9. The desk is .
10. The woman is .

 Bƣớc 3: Nghe câu và chọn đáp án đúng.


1. A. People are .
B. Some people are .
C. People are .
D. People are .
E. People are .

2. A. There is on the table.


B. Both people are .
C. She is the paper.
D. People are standing .
E. The man is .
TOEIC FOCUS ---- TOEIC BUILDING-----Smart Home for Toeic

3. A. There are some flowers _ .


B. The men are .
C. are standing behind the
podium.
D. The men are .
E. One of the men is .

4. A. The man .
B. The man is his sunglasses.
C. The bag has been placed .
D. He is at the beach.
E. There is .

5. A. at the moment.
B. The lights are .
C. The chairs are .
D. Some seats are set .
E. Attendees are .

6. A. All the men are .


B. They are .
C. They‟ve just .
D. They are .
E. They are .

11
TOEIC FOCUS ---- TOEIC BUILDING-----Smart Home for Toeic

LESSON 2: PRACTICE
Practice 1

12
TOEIC FOCUS ---- TOEIC BUILDING-----Smart Home for Toeic

Practice 2

14
TOEIC FOCUS ---- TOEIC BUILDING-----Smart Home for Toeic

15
Practice 3
TOEIC FOCUS ---- TOEIC BUILDING-----Smart Home for Toeic

Practice 4

19
TOEIC FOCUS ---- TOEIC BUILDING-----Smart Home for Toeic

20
Practice 5
Practice 6
Practice 7
TOEIC FOCUS ---- TOEIC BUILDING-----Smart Home for Toeic

29
TOEIC FOCUS ---- TOEIC BUILDING-----Smart Home for Toeic

PART 2: QUESTIONS AND RESPONSES


LESSON 1: WH-QUESTIONS

I. Question with Who / Whose / Whom

1. did you ask about ?

A. The should be .

B. Dave Rowles _ .

C. It will be in a .

2. office will be mine?

A. It is the .

B. No, it‟s Mr. Ford‟s .

C. Your will be next.

3. put up the notice on the ?

A. At about .

B. It was Elaine from .

C. To everyone about it.

4. should I talk to about the ?

A. You can use if you want.

B. Yes, he‟s the one property.

C. You can it with me.

5. has the committee selected ?

A. No, you should this one.

B. In the near future.

C. They haven‟t yet.

6. was that for?

A. Helen Parker.

30
B. Late at.

C. It was from the .

7. car are you going to ?

A. No, .

B. My colleague‟s.

C. I should by tomorrow.

8. going to when you’re away?

A. Yes, she will do that.

B. Neither do I.

C. will decide that.

II. Questions with Where

1. Where did you that handbag?

A. Yes, it is quite .

B. That‟s fine. I will .

C. It was a .

2. Where can I buy some ?

A. You very good.

B. Try the .

C. I don‟t anymore.

3. Where are the for the ?

A. I being there a bit early.

B. There are still a few .

C. On the over there.

4. Where is the report from the ?

A. It won‟t be until this afternoon.


B. to my desk.

C. Yes, I‟ve the report.

5. Where do you keep the _ ?

A. There are in the .

B. Not too , I hope.

C. It‟s been and away.

6. Where was the held last month?

A. At our Middleburgh .

B. We met a .

C. It was held from to .

7. Where do you have to go my ?

A. There is a of 60 pounds.

B. Check the in the .

C. To the over there.

8. Where are they going to the ?

A. They want a higher .

B. They‟re still several .

C. Are you ?

III. Questions with When

1. When will the begin work?

A. Yes, he very hard.

B. For the last .

C. On the next month.

2. When will the be over?

A. I wish I could you.


B. and .

C. The is over now.


3. When did his job at the _?
Mark

A. To some fruit.

B. A weeks ago.

C. He works from to .

4. When is this problem going to ?

A. I‟m not it.

B. At the next meeting.

C. Yes, this is a really .

5. When do you expect the back from his ?

A. At today.

B. No, not .

C. To an .

6. When will the begin?

A. The is this weekend.

B. After .

C. It‟s been now.

7. When will the next of the newsletter be ?

A. At the .

B. I could it to you .

C. Next .

8. When do you want to ?

A. In .

B. Not until .
C. Yes, I already him.

IV. Questions with What / Which


1. What is the for this computer?

A. It‟s the outlet .

B. The is computerized.

C. There are 5 .

2. What do you of the new ?

A. On the floor.

B. We‟ll have a lot .

C. It‟s a big over the old one.

3. Which one of these will I have to ?

A. Don‟t to at the bottom.

B. The one marked .

C. Yes, the firm is our .

4. What time would it be now?

A. Her flight will 7.

B. It‟s a very _ center.

C. The time difference is hours.

5. What should I do with the after I ?

A. Leave them with my .

B. You should it by tomorrow.

C. the software whenever you want.

6. What does Mr. Lee think of ?

A. He said it was quite .

B. Probably at of this month.


C. I don‟t think he .

7. Which of you read this ?

A. I prefer .

B. Maria did.

C. pages.

8. What does the new look like?

A. It it will be next month.

B. She‟s looking at her .

C. She is and glasses.

V. Questions with Why

1. Why was the ?

A. It is next week.

B. Yes, I‟ll it.

C. We haven‟t for it yet.

2. Why did you here so today?

A. Yes, I will of it later.

B. I got in traffic.

C. The rate is a bit .

3. Why wasn’t this report to Mr. Dall?

A. Ok, let‟s it.

B. They‟re with the plans.

C. Some have to be made.

4. Why has the been ?

A. Only for a _ .

B. Because of a _.
C. The is Chicago.

5. Why did the company ?

A. To lower rates.

B. To the district.

C. Moving were very high.

6. Why weren’t you at the company over the ?

A. No, I‟m not this weekend.

B. Something .

C. before noon.

7. Why is Mr. Donaldson so the board meeting?

A. We are very with his .

B. Because I‟m _ about .

C. He‟s going to be .

8. Why was the sent back?

A. The address was .

B. We will never there .

C. By morning.

VI. Questions with How

1. How can I to this room?

A. No, I don‟t have to it.

B. Ask Miranda, the .

C. For the weekly .

2. How many are we planning to today?

A. Everything you here.

B. They should today.


C. That might be .

3. How much is the ?

A. There is .

B. Let me the price.

C. It‟s ten after .

4. How was the ?

A. in the morning.

B. It was .

C. It sure .

5. How do you like the new video ?

A. It seems much .

B. I think the will be here .

C. No, I think so.

6. How long has Terry been ?

A. A month as of .

B. He quit over a with the .

C. It won‟t him .

7. How did you get ?

A. Just a on Burton Avenue.

B. The drive _ 45 minutes.

C. There was a on the company .

8. How are the for next year?

A. The raise has been .

B. Things are looking quite .

C. The is going to be pretty _.


Practice Test

1. (A) (B) (C) 6. (A) (B) (C) 11. (A) (B) (C) 16. (A) (B) (C)
2. (A) (B) (C) 7. (A) (B) (C) 12. (A) (B) (C) 17. (A) (B) (C)
3. (A) (B) (C) 8. (A) (B) (C) 13. (A) (B) (C) 18. (A) (B) (C)
4. (A) (B) (C) 9. (A) (B) (C) 14. (A) (B) (C) 19. (A) (B) (C)
5. (A) (B) (C) 10. (A) (B) (C) 15. (A) (B) (C) 20. (A) (B) (C)

1. How many class are going on the ?

A. Roughly .

B. Yes, they .

C. No, not .

2. What did you Tuesday from the ?

A. No, it was .

B. He me a .

C. I them.

3. Who was when the was away?

A. For about _ .

B. Her .

C. I believe it was.

4. What will the be ?

A. I have not to her.

B. It will about an .

C. earnings for next year.

5. Where can I Ms. Hodge in ?

A. She‟s the .

B. and go through the second .


C. Yes, she‟s a very .

6. How will I if it is Mrs. Fuller?

A. Sidney knows the .

B. I she did.

C. She‟ll have on a .

7. How come you’re to Brussels on ?

A. Have a .

B. Yes, on morning.

C. To meet with some .

8. Why is that ?

A. About .

B. The is on.

C. Use a corkscrew to it .

9. How often does the ?

A. On the .

B. a month.

C. For hour.

10. Which of these should I ?

A. The to the far .

B. Your are excellent.

C. Yes, we‟re used to hours.

11. Where is the ?

A. She .

B. I think Ms. York it.

C. Sometime this .
12. How are they going to the to France?
A. I used to France.

B. For about _ .

C. By .

13. How long will it to the financial ?

A. A of months.

B. We‟re a firm to assist.

C. It‟s being given .

14. When will the be complete?

A. He is a worker.

B. Not for month or .

C. You should this first.

15. What type of are you ?

A. Yes, I should _ it .

B. I‟m in .

C. We should on it more.

16. Why did you this way to Karen’s ?

A. It‟s quicker during .

B. Because she‟s highly .

C. No, I‟ve never there.

17. When did the give his for the ?

A. Yes, he to prove it.

B. Because the wants me to.

C. While he was here today.

18. Which part of the was most to you?


A. He a of it.

B. It was a party.

C. The building .

19. How is the sound ?

A. It‟s much than our one.

B. We‟re on a new .

C. That sounds like a .

20. Who’s for taking care of the flight ?

A. To Boston.

B. Ms. Elliot that.

C. I am to that day.
LESSON 2: YES-NO QUESTIONS

1. Các câu hỏi Yes / No cơ bản

a. Câu hỏi với trợ động từ và động từ khuyết thiếu

1. Has the fax from the yet?

A. No, I haven‟t anything.

B. Yes, I to the .

C. On the of the new building.

2. you see much between those two ?

A. They look to me.

B. I a different .

C. They liked both .

3. Will Mr. Kyle his this week?

A. A very course.

B. He is a great _.

C. No, not until .

4. Did you the latest news ?

A. My hasn‟t been yet.

B. Yes, in minutes.

C. No, what‟s ?

5. Should I the ?

A. the door.

B. Yes, the could use some .

C. I‟d like to an .

6. Do you have another of the ?

A. Two other .
B. afternoon.

C. It‟s in my .

7. Have you been find anyone to Peter?

A. he‟s going to our New York .

B. The of hasn‟t made a _ yet.

C. I‟ve been _ it for 2 hours.

8. Do you know if the we had ordered meet industry


?

A. Because the is .

B. this week.

C. Yes, we the best that were .

b. Câu hỏi với động từ be

1. Are you going to for the event?

A. The show is to be a .

B. Yes, I‟ve already .

C. You have to this weekend.

2. Is it possible to the early?

A. The bank at 5.

B. A is due next .

C. Not an fee.

3. Was the ?

A. For the most _, yes.

B. We just the .

C. Not until .

4. Is there more me to do after I’m with this?


A. I , too.

B. Yes, .

C. No, I today.

2. Câu hỏi phủ định

1. Don't you think we should Mr. Jeffers about the ?

A. No, I don't that's him.


B. Yes, I think .
C. I him to be time.
2. Isn't Rick to the company this evening?

A. He is next week .
B. No, he's gone on a .
C. I've to go to the party.
3. Didn't Kevin already staff ?

A. In the main room.


B. It won't _ .
C. Yes, but many are still .
4. Mrs. Chiu for the day yet?

A. Yes, she near here.


B. No, she's next .
C. No, she usually until .
5. Isn't that car a bit ?

A. For days.
B. We aren't very far.
C. Yes, but we don't any other .
6. Shouldn't we some time to the annual report?

A. No, I am not a speaker.


B. I'm this .
C. Because it's almost .
7. Haven't you been to a new yet?

A. The new looks .


B. It's hard to _a applicant.
C. Yes, he is to the .
8. Wouldn't the look more if the pictures were and
?

A. Let's hang it next to the .


B. Yes, we should it first.
C. Well, let's if it .
3. Câu hỏi gián tiếp

1. Do you know the owner of is?

A. The is downstairs.
B. No, I don't know it is.
C. I to Karen.
2. Did Mr. Long why the is being ?

A. Because of .
B. It's not being at all.
C. Yes, at the .
3. Can I ask you the bus stop is around here?

A. There is one at the .


B. It every half hour.
C. No, it's .
4. Can you me how to use ?

A. On the .
B. Sure, just _ me a .
C. copies.
Practice Test

1. (A) (B) (C) 6. (A) (B) (C) 11. (A) (B) (C) 16. (A) (B) (C)
2. (A) (B) (C) 7. (A) (B) (C) 12. (A) (B) (C) 17. (A) (B) (C)
3. (A) (B) (C) 8. (A) (B) (C) 13. (A) (B) (C) 18. (A) (B) (C)
4. (A) (B) (C) 9. (A) (B) (C) 14. (A) (B) (C) 19. (A) (B) (C)
5. (A) (B) (C) 10. (A) (B) (C) 15. (A) (B) (C) 20. (A) (B) (C)

1. Do you have the receipt?

A. It's dollars.
B. No, I it in my .
C. Because I'm cash.
2. Isn't that open this time of ?

A. No, it closes at .
B. We've _ of gas.
C. No, I didn't it yet.
3. Have you updating your ?

A. The site is regularly.


B. We tomorrow.
C. It's nearly _.
4. Mrs. Winders here?

A. No, I have to until late at .


B. Yes, I have here year.
C. No, she a month .
5. Is there any ?

A. There's more on the .


B. Thanks, but I'm too _ .
C. I had some last night.
6. we send out a to staff members?

A. We received the by .
B. It is in the _.
C. That probably won't be .
7. Are you with in this area?

A. No, it isn't my .
B. This is my time her.
C. Yes, he is a good .
8. Don't you think this needs to be further?

A. A number of came .
B. is set for November 21.
C. We're already some major .
9. Sorry, but haven't I you ?

A. Yes, I so.
B. No, it's not .
C. Let's tomorrow.
10. Can you someone to next weekend?

A. Take any _ you want.


B. We have children.
C. I'll ask my .
11. Aren't you coming to
with us?

A. He's right _ .
B. The broke sales .
C. Sorry, but I have a .
12. Do you know Ms. Chao is having the ?

A. In Room .
B. Yes, to the .
C. It start at o'clock.
13. Will you the staff on how to the new ?

A. It will be tomorrow.
B. That's _ the .
C. I think they're _ the .
14. Did Albert to work out the with the heater?

A. Yes, he already of it.


B. It is a in here.
C. Well, we could together.
15. Is Mrs. Sampson you?

A. Yes, I have an for .


B. She'll be for a .
C. Yes, I've
her before.
16. Hasn't anyone
to in for Mr. Bates?
been

A. Yes, please .
B. He's been _ here for years.
C. yet.
17. Did they why the was ?

A. Our won't be here until .


B. Yes, we don't have to .
C. There was a .
18. Do you have any time up?

A. It's for the upcoming .


B. We to the .
C. Sometime .
19. Have any of the looked at the ?

A. Please _ for everyone.


B. A number of .
C. Lisa will in just a or .
20. Are you the franchising ?

A. I not to .
B. Many companies will be .
C. Unfortunately, I'm .

LESSON 3: OTHER TYPES OF QUESTIONS

1. Alternative questions - Câu hỏi lựa chọn


1. Will the be or ?
A. Yes, they‟re with my .
B. Please to be there by .
C. I it‟s on .
2. Would you me to you , or do you like
?
A. Your is quite .
B. than a away.
C. I would a_ .
3. Would you like a or an ?
A. is .
B. It‟s on an .
C. the .
4. Do you it , or would you taking it ?
A. There is a small .
B. I‟ll it up.
C. Go and it.
5. Do you get for your magazine or in ?
A. We pay .
B. Here‟s a of my .
C. I can him _ .
6. Will the client or the for his ?
A. We can him there.
B. It‟s in the leasing .
C. He is of buying.
7. Would you like to have or today?
A. It was sent to _ .
B. The Hong Kong .
C. , I‟d have today.
8. Did you the to the , or are you
going to call a instead?
A. Yes, it is to rain today.
B. No, I‟ll see if I can do it .
C. Yes, I think it‟s that.
2. Tag questions - Câu hỏi đuôi
1. You a new , didn’t you?
A. Yes, I need to .
B. Yes, it should be soon.
C. No, she‟s not _ for the .
2. Senior are for a , aren’t they?
A. Yes, she‟s for the .
B. Only 3 PM.
C. Sorry, I didn‟t them.
3. You didn’t out the yet, did you?
A. No, I had to the first.
B. The has just .
C. Yes, I play .
4. Mr. Lee is the next in line to a , isn’t he?
A. More than a .
B. No, the isn‟t very .
C. Yes, he‟s to get one now.
5. Jack never to come on for our , does he?
A. I‟d to him about it.
B. No, I didn‟t there was a .
C. Yes, we are going to 9.
6. You a that looks like this one, don’t you?
A. No, I don‟t her.
B. Yes, everyone to one.
C. No, it was at the .
7. You’re to Dr. Henson’s party, aren’t you?
A. Yes, but I be on .
B. Yes, he is going .
C. the staff.
8. Michael Parkinson’s about is very
, don’t you ?
A. He didn‟t the meeting.
B. I have not had a to it yet.
C. Yes, we‟ve finally an .
3. Statements - Câu nhận định
1. Maybe you should get a .
A. I have one .
B. The is for _ .
C. I there are a .
2. I just want to you that it’s going to be very on the
.
A. I need in my .
B. Thanks, I‟ll some .
C. We are going to .
3. We’re up a new .
A. No, it‟s in the 10th .
B. , we really _ one.
C. The has at the .
4. A just a of on the table.
A. A .
B. The table in of the .
C. I‟ll someone it up.
5. I really that nice on you.
A. You should the .
B. , I got it .
C. Sure, you can _ it.
6. not make a until we from Ted.
A. We need his .
B. I the on the .
C. Yes, I it in .
7. I’m Henry Davidson the .
A. Yes, I for it.
B. We will to get in with him.
C. He made a .
8. I’m sorry, but I can’t this evening’s after all.
A. There was a .
B. Do you have to again?
C. I don‟t what to either.
4. Requests, invitations, offers - Yêu cầu, lời mời, lời đề nghị
1. Would you if I the ?
A. The closes at 6.
B. Not .
C. It‟s the by the .
2. Could you these ?
A. No , would you them there?
B. I the .
C. No, I was yesterday
3. Why don’t we the members our ?
A. Yes, on the .
B. All right. I‟ll _ them in .
C. No, I didn‟t to at it yet.
4. this for me, will you?
A. I .
B. I I can that for you.
C. He it this .
5. Would you be in our ?
A. , but I already one.
B. Sure, in the .
C. No, I‟ve already _ it.
6. don’t we have a for the ?
A. That would be a of .
B. I really a good .
C. That like a .
7. along to the Newton Philharmonic
?
A. The should about 3 .
B. That be .
C. No, for 2 .
8. Would you me to a for Mr. Johnson?
A. He has a .
B. Yes, I him very .
C. Thanks, but I‟ll again .
Practice Test

1. (A) (B) (C) 6. (A) (B) (C) 11. (A) (B) (C) 16. (A) (B) (C)
2. (A) (B) (C) 7. (A) (B) (C) 12. (A) (B) (C) 17. (A) (B) (C)
3. (A) (B) (C) 8. (A) (B) (C) 13. (A) (B) (C) 18. (A) (B) (C)
4. (A) (B) (C) 9. (A) (B) (C) 14. (A) (B) (C) 19. (A) (B) (C)
5. (A) (B) (C) 10. (A) (B) (C) 15. (A) (B) (C) 20. (A) (B) (C)

1. don’t we start the at 8 PM?


A. No, we .
B. was here.
C. Can we start a ?
2. Would you or ?
A. We‟re going to .
B. one is larger.
C. The is right over there.
3. The are in the , right?
A. Yes, she is the new .
B. I don‟t need this now.
C. No, I just them .
4. Mrs. Singer, I need you to to .
A. I‟ll call my right away.
B. The flight was terrible.
C. I one more.
5. Would you like to see our ?
A. No, just some please.
B. I didn‟t them.
C. I don‟t anyone.
6. You haven’t Harry today, ?
A. He‟s in the .
B. I will do it .
C. We have before.
7. Should we the house , or would be better?
A. It‟s very .
B. Yes, .
C. I .
8. Can you take a of us this ?
A. Yes, do I just this ?
B. I I should take a .
C. Please two .
9. Would you like a with your ?
A. I need to it, too.
B. By the end of next .
C. Thanks, but I can .
10. That’s the I got my new .
A. No, a .
B. They seem to have the in town.
C. You can get one .
11. Should this be to the or the department?
A. There are of at the moment.
B. Let the manager of it.
C. It‟s better to take a .
12. We should be some new soon, shouldn’t we?
A. I have to get this .
B. Mr. Jeffers our .
C. Not next month.
13. Has this been , or should I use a one?
A. It‟s now.
B. We‟re having the soon.
C. Get the while you‟re there.
14. Would you mind this for ?
A. Just give me .
B. You‟re to do the .
C. I‟ll pay by _ .
15. These are hard to .
A. Which are you going?
B. I‟m on the _ .
C. I can‟t them , either.
16. Are you going to the , or do you me to it?
A. There wasn‟t a .
B. His was .
C. I can do it, if you .
17. We should all the employees’ in our main .
A. Yes, the is quite good.
B. Yes, there have been many .
C. No, the has not been yet.
18. You’re not going to be next , are you?
A. Yes, it‟s just around the .
B. No, that‟s I go to .
C. Yes, good to me.
19. Can you me the by the end of the day?
A. Yes, all long.
B. I‟ll to that for you.
C. It‟s going to soon.
20. You the new Mark Jennifer , didn’t you?
A. Yes, I it on .
B. No, he‟s been .
C. I need for my .
LESSON 4: PRACTICE

1. (A) (B) (C) 7. (A) (B) (C) 13. (A) (B) (C) 19. (A) (B) (C) 25. (A) (B) (C)
2. (A) (B) (C) 8. (A) (B) (C) 14. (A) (B) (C) 20. (A) (B) (C) 26. (A) (B) (C)
3. (A) (B) (C) 9. (A) (B) (C) 15. (A) (B) (C) 21. (A) (B) (C) 27. (A) (B) (C)
4. (A) (B) (C) 10. (A) (B) (C) 16. (A) (B) (C) 22. (A) (B) (C) 28. (A) (B) (C)
5. (A) (B) (C) 11. (A) (B) (C) 17. (A) (B) (C) 23. (A) (B) (C) 29. (A) (B) (C)
6. (A) (B) (C) 12. (A) (B) (C) 18. (A) (B) (C) 24. (A) (B) (C) 30. (A) (B) (C)

1. is the ?
A. To some .
B. them there.
C. Just the .
2. is your ?
A. The was .
B. At the .
C. This at 3.
3. was this from San Diego?
A. It on time.
B. By .
C. In San Diego _ .
4. Would you like to a of our new
?
A. No, it‟s quite .
B. The was quite .
C. Sure, it looks .
5. Are you it’s to in this ?
A. Let‟s have the .
B. Of , she did.
C. As as we‟re .
6. Don’t you we should for from the main
?
A. I there week.
B. We already have .
C. She it was .
7. Will you be or next week ?
A. That‟s Mr. Ghent.
B. It‟s too to _ my .
C. I‟m going to the .
8. Would you like to to a tonight?
A. people will .
B. I I could, but I some _ to do.
C. It‟s a way from your .
9. Should we the today, or get new ?
A. There is of office .
B. It‟s your to the .
C. Let‟s of the _ first.
10. We should within an .
A. I‟ll be by then.
B. I it over there.
C. No, it about 2 .
11. can’t you at the time?
A. It should this .
B. It was a to _ you.
C. There has been an .
12. Mr. Norton has the , hasn’t he?
A. He the and was .
B. No, he‟s been with other .
C. If you to.
13. don’t you about that next when the
is ?
A. That would be for me.
B. We are the .
C. Yes, I‟d like to have a .
14. Should we have this , or on ?
A. Let‟s have .
B. would be for me.
C. Yes, the will be .
15. should this be sent?
A. By at the .
B. It is 25 long.
C. To the office.
16. will be with the on ?
A. It hasn‟t been _ yet.
B. It‟ll be about _ .
C. He will here at 7 PM.
17. are we going to the ?
A. 3 ago.
B. We‟re a company.
C. I the .
18. is in of the members?
A. is very .
B. A service will be .
C. That would be the .
19. can I in with you?
A. this will be .
B. I would like to again.
C. He is an business .
20. will they be the ?
A. It will start next month.
B. costs are going up.
C. A bank will provide a .
21. of have you least ?
A. ads haven‟t very .
B. I didn‟t to the today.
C. Thanks. You‟re very .
22. Don’t we have an next month?
A. Yes, it‟s going to be a .
B. No, we‟ll get the next week.
C. Yes, the was on the floor.
23. You will about to there, right?
A. No. I‟ve already _ it.
B. I‟ll there by .
C. like an or so.
24. was Mrs. Dennis given a ?
A. It is on a .
B. She needs _ to the .
C. She her last .
25. Do you have any rooms for the next ?
A. We need to a .
B. No, I won‟t be to go.
C. Those are already all up.
26. will be at this morning’s ?
A. The has a lot of .
B. Some new .
C. From to .
27. This is for the .
A. I‟ll it for you.
B. The has been .
C. Please here.
28. We’re going to , aren’t we?
A. The has been well.
B. I‟m up the _ right now.
C. Yes, we‟re right now.
29. Do you know that is due?
A. On the desk.
B. the of the .
C. She‟ll that.
30. Wasn’t that a ?
A. Yes, I‟ve already a .
B. I wouldn‟t that.
C. No, it‟s in the .
PART 3: SHORT CONVERSATIONS
LESSON 1: TYPES OF QUESTIONS
I. Overview questions - Câu hỏi tổng quát
1. Câu hỏi về chủ đề hoặc mục
đích Typical questions:
What are the speakers/ they discussing?
What are the speakers/ they talking about?
What is the conversation about?
Why is the woman calling?
What is the purpose of Mark‟s phone call?
Example:
Q. What are they talking about?
A. The weather forecast
B. A catering service
C. A change in plans
D. Employee training
Script:
W: It doesn‟t look like the weather is going to .
M: Yes, if the rain keeps up .
W: ? There is at least one conference room in
the convention center that‟s still available
M: That‟s true, and I think it‟s all of us. I‟ll
call the caterers and let them know that we may _ .
2. Câu hỏi về địa điểm và nghề nghiệp
Typical questions:
- The speaker’s or speakers’ occupations
Who most likely is the man/ woman?
Who is the woman calling?
Who is the man most likely talking to?
Who is the woman speaking
with? Who are the speakers?
- Locations
Where does this conversation (probably) take
place? Where most likely are the speakers?
Example 1:
Q. Who most likely is the man?
A. A police officer
B. A security officer
C. A lawyer
D. A computer technician
Script:
M: . How can I help you?
W: Hello, this is Keiko Yamamoto from the legal department. My
and I can‟t . Do you think you
could come by and have a look at it?
M: No problem. Where is your office?
W: I‟m on the 14th floor in Room 1492. It is the
on your right.
Example 2
Q. Where does this conversation most likely take place?
A. In a theater
B. In a museum
C. In a bookstore
D. In a travel agency
Script:
M: Hello. I‟m trying to by Angelina Aniston.
Do you know where I might find it?
W: Of course. She writes , doesn‟t she?
M: That‟s right. are mostly about life in
ancient Egypt and northern Africa.
W: Alright, then I think you‟ll find on the
second floor next to the .
Từ vựng về nghề nghiệp
Bookkeeper Sales representative Locksmith
Construction worker Security officer Surgeon
Coworker Shift manager Technical support person
Electrician Bank teller Interpreter
Factory worker Cashier Journalist
Interviewer Client (= customer) Photographer
Job applicant Fitness instructor Translator
Maintenance worker Housekeeper Flight attendant
Mail carrier Librarian Hotel clerk
Operator Plumber Restaurant server (=waiter)
Office assistant Physician Travel agent
Potential employee Parking attendant Tour guide
Receptionist Newsstand vendor Repairman
Salesperson (sales clerk) Musical performer

Từ vựng về địa điểm và các khái niệm liên quan


Restaurant Order Table Customers
Dessert Seats Kitchen Staff
Bill Reservation Plates
Eat Menu Silverware
Food Recipe
Waiter Dish

Hotel Room Room service Suitcase


Key Check in Register
Arrange for a shuttle Check out Conference
Front desk Lobby

Bank Loan Balance Overdraw


application Account Transfer
Savings account Withdraw Identification
Deposit
Post office Ship Express mail Package
Box Stamps Standard delivery
By air Send Overseas
Surface mail Weigh
TOEIC FOCUS ---- TOEIC BUILDING-----Smart Home for Toeic

Dentist’s office Filing Dentist Scaling


Teeth Cavity

Pharmacy Prescription Medicine Pill


Tablet Dose

Airport Gate Flight Delay


Information Departure Check the bag in
desk Luggage Flight information
Board Passenger

Store Try on Pay by cash / check Show


Size / card Refund
Model Gift-wrap I‟m looking for ~
Discount Shop window Can you tell me
Display Counter Where can I find ~?

Bookstore Bestseller Shelves On the third floor


Section Aisle
Journal Novel

Library Books Lost item drawer Bookshelf


Check out Reshelve

Travel agency Brochure Make a reservation Travel agent


Trip Ticket Itinerary

Real estate agency Property For sale Move in


Rent Site
Landlord Resident

Theater Seats left Book / Reserve Leave early


Tickets Wait in a line
Sold out Audience

Practice
1. What are the speakers discussing? 5. What is the conversation mainly about?
A. Travel plans A. A service charge
B. A missing document B. Office hours
C. Scheduling a meeting C. A shipping company

65
D. A new client D. Sending a letter
2. Where most likely are the speakers? 6. Where most likely are the speakers?
A. At an airport A. At an office supply store
B. At a travel agency B. At a post office
C. At a convention center C. At an airport
D. At a train station D. At a restaurant
3. What are the speakers talking about? 7. Why is the man calling?
A. A musical performance A. To make a lunch appointment
B. A delayed shipment B. To arrange an interview
C. A recently released CD C. To request a donation
D. A computer error D. To cancel a meeting
4. Who most likely is the man? 8. Who most likely is the man speaking to?
A. A store clerk A. A reporter
B. A customer B. A financial consultant
C. A musician C. A receptionist
D. A warehouse manager D. An interviewer

II. Detail questions - Câu hỏi chi tiết


a. Questions with What
Typical questions
- The speaker’s or speakers’ problem or concern
What is the woman‟s problem?
What is the man concerned about?
- What the speaker(s) has (have) talked about
What does the man say about
Mary? What does the woman
suggest?
What does the man recommend?
- What the speaker(s) will do next
What will the speakers do next?
What will the woman probably do next?
- Specific information
What is scheduled to take place next week?
What was the man not able to do?
- Interpreting what the speaker(s) is (are) implying
What is suggested about the event?
TOEIC FOCUS ---- TOEIC BUILDING-----Smart Home for Toeic

What does the man suggest about the manager?


Example 1
Q. What does the man suggest?
A. Taking the subway
B. Paying the bill
C. Calling the restaurant
D. Visiting the woman‟s office
Script
W: Regis Copy Center. This is May North. Can I help you?
M: Ms. North? My name is Scott Grant at Garibaldi‟s restaurant. I
this afternoon and
from a

I found inside.
W: That‟s wonderful news! I thought I had on
the subway.
M: Well, it is here at the . Actually, I know where Regis Copy Center is. I can
on my way home, if you‟d like.
Example 2:
Q. What does the woman suggest about the Zurich branch?
A. It is the company‟s headquarters
B. It is the company‟s smallest branch
C. It is expanding its offices
D. It has not opened yet.
Script:
W: Did you hear that Ms. Hammer is to
Zurich? She was an at our
I just heard about it this afternoon.
M: No, I hadn‟t heard about it. does she ?
W: In about 3 weeks. They want her in Zurich before the
.
b. Questions with Why
Typical questions
68
Why was the meeting canceled?
Why does the woman want to reschedule her appointment?
Why is the man concerned?
Why are the speakers
pleased? Why is John going to
Athens? Why is the man late?
Why is Ron unable to attend the seminar?
Example:
Q. Why does the man want to leave early?
A. He needs to see a family member.
B. He is going to see a doctor
C. He has an appointment at a bank
D. He will be leaving for vacation
Script:
M: Ms. Greyson, I just wanted to you that I need to

early today. I to meet my


this afternoon.
W: Of course, I remember. But I need you to the
first. I have to them to the
at Monday‟s meeting.
M: Certainly, I‟ve been on them all morning. I should have them
after lunch. I‟ll them on your
before I go.
c. Questions with How
Typical questions
- Methods/ means
How will the man go to the conversation?
How will the woman contact the manager?
How will the company increase its sales?
- Duration/ frequency/ quantity
How long will the man stay in Paris?
How often is the meeting held?
How much of a discount will they get?
Example:
Q. How will the speakers most likely get to the hotel?
A. By car
B. By bus
C. By subway
D. On foot
Script:
M: According to this
the is being

in the Grand Moreno Hotel on West 2nd. It starts at 7, so we‟d better


if we don‟t want to be _ .
W: Why don‟t we ? It‟s be than
the .
M: It‟s the of , and will be
. Plus, I‟m pretty sure the Moreno doesn‟t have a
. It‟ll be to
nearby.
W: Let‟s on the then. The hotel is to the 2nd
Street stop.
d. Questions with When/ Where/ Who
Typical questions:
- When:
When are the speakers going to
meet? When is the new deadline?
When did the woman order the brochures?
What time can also be used to ask about time?
What time is the woman‟s flight?
- Where
Where does the man want to go?
Where will the meeting be held?
Where did the man leave his coat?
- Who
Who will order the stationery?
Who is going to resign next month?
Who is the woman going to meet?
Example:

Q1. Who is going to meet with the chairman?


A. Gabrielle
B. The accounting manager
C. A reporter
D. A project manager
Q2. When will the woman probably submit her analysis?
A. On Thursday
B. On Friday
C. On Monday
D. On Tuesday
Q3. Where is the chairman now?
A. At the accounting department
B. At a convention center
C. At a branch office
D. At a conference room
Script:
M: Gabrielle, do you think you‟ll be able to the by
Thursday night? The wants to have all the
by Friday morning so he can
them to the

for the review.


W: Well, I have a really , but I might be able to it by
Thursday.
M: The is at the this week, so there won‟t be a meeting
.
W: Thanks, but there shouldn‟t be a . I‟ll do my best to on
Thursday.
Practice
1. Why does the man not want to 7. What problem does the woman
help Veronica? have with Dreamsoft?
A. He is busy with an important report A. They submitted the wrong report
B. He has to attend a special conference B. Their software has some technical
C. He is resigning from the company problems
D. He will be on leave that day C. They do not want to amend the
2. What does the man want the woman contract terms
to do? D. Their representatives are difficult to
A. Assist him with job interviews contact
B. Help him to prepare a report 8. When will the contract most likely
C. Attend an interview with him be signed?
D. Accompany him to the boss‟s office A. On Monday
3. How often does the quality control B. On Wednesday
report come out? C. On Friday
A. Every week D. On Saturday
B. Every month 9. What will the woman probably do next?
C. Every two months A. Call an emergency meeting
D. Every three months B. Negotiate with another company
C. Write a report for the boss
4. Where do the speakers work? D. Talk to the Dreamsoft staff
A. In Chicago
B. In Paris
C. In London
D. In New York
5. What did the woman recently do?
A. Presented a paper at a conference
B. Started a new position
C. Took a business management class
D. Shipped her products abroad
6. How long has woman been waiting
for her cell phone to be connected?
A. One day
B. Two days
C. Four days
D. One week

LESSON 2: CONVERSATIONAL SITUATIONS

1. Office and corporate property


*** Common situations:
- Office
- Corporate property
*** Typical questions:
a, Overview questions:
1. The topic/ purpose of a conversation
What are the speakers discussing?  Plans for a meeting
2. The speaker‟s or speakers‟ occupation/
position Who most likely is the man?  A maintenance
employee
3. The location of a conversation
Where does the conversation probably take place?  In an office
b, Detail questions
1. The main problem/ concern
What problem does the man mention?  The printer is broken
2. A specific suggestion
What does the man offer to do?  Deliver some contracts
3. A specific piece of information
What do the speakers say about the product?  It‟s not selling well
4. A particular department/ company
What kind of company does the woman work for?  A manufacturing company
Example:
Q1. What problem are the speakers discussing?
A. The copy machine is being repaired
B. The meeting has been postponed
C. The man‟s computer is broken
D. The fax is not working
Q2. When does the man need to finish the proposal?
A. Today
B. Tomorrow
C. Next Monday
D. A week later
Q3. What does the woman offer to do?
A. Call the technician
B. Deliver a document
C. Cancel a meeting
D. Phone another office
Script:
M: Have you talked with the . I need to e-mail this to the
and the isn‟t .
W: I with them this morning, but they said that they wouldn‟t be able
to send a to look at it .
M: Oh, no. The for the is this afternoon. I‟m going to have to
it to them myself, but I don‟t have any time today.
W: Well, I have a meeting later near the
I can
for you if you want.
Paraphrasing practice
1. Finish a summary for the committee 
2. The estimate won‟t be complete until Friday 

3. Check to see if the report looks OK 


4. A lot of information was wrong 
5. It went to the audit department by mistake 
6. Expect some documents 
7. She is going to stop by to get the packet 
8. Reschedule the meeting 
9. The copier keeps breaking down 
10. The generator is malfunctioning 
11. The machine was not plugged in 
12. The power‟s out 
13. I have too much work to do 
Practice
1. What are the speakers discussing? 7. What does the woman ask the
A. A branch of the company man to do?
B. A medical exam A. Write a proposal
C. Some tax documents B. Call a client
D. A new address C. Organize a meeting
2. Why is the woman leaving her office? D. Make a presentation
A. She has to meet her accountant 8. What is the purpose of the meeting?
B. She has to go to a hospital A. To resolve a technical issue
C. She has a meeting on parker street B. To discuss the latest financial report
D. She needs to buy a fax machine C. To solve a problem with the
3. What does the woman say she will management
do tomorrow? D. To talk about a possible deal
A. Read some documents 9. What does the management want
B. Visit her doctor to know?
C. Send the man a fax A. Whether the acquisition is practical
D. Return to the office B. When the meeting will be held
------------------------------------------------------------ C. When the takeover will take place
4. Where does the man work? D. Which technologies they will gain
A. At an Internet service company
B. At an engineering company
C. At a laundry service
D. At a fabric company
5. Why did the serviceman not come in
the morning?
A. He was busy with another call
B. He was too ill to come to work
C. He was help up by an emergency
D. He was delayed by a server problem
6. When is the replacement technician
supposed to arrive?
A. 8:00 a.m
B. 9:30 a.m
C. 10:00 a.m
D. 12:00 p.m
----------------------------------------------------------------

2. Human resources
*** Common situations:
- Recruiting / Hiring
- Training and promotion
*** Typical questions:
a, Overview questions
o The topic / purpose of a conversation
1, What are the speakers discussing?  Hiring new employees/ a job interview
o the speaker‟s or speakers‟ occupation/ position
2, Who most likely are the speakers?  New managers
o The setting of a conversation
3, Where does this conversation probably take place?  In a job interview
b, Detail questions
o What will take place
1, What will take place next week?  A training seminar
o What position is applied for
2, What position is the woman applying for?  Sales representative
o When something will take place
3, When will the interview probably take place?  Next Thursday
o How documents are submitted
4, How will the man submit his resume?  By mails
Example

Q1. Where does this conversation probably take place?


A. In a school
B. In a conference room
C. At a job interview
D. At a department meeting
Q2. What is the woman interested in?
A. Writing articles
B. Coming up with ideas
C. Becoming a manager
D. Scheduling an appointment
Q3. What does the man offer to arrange?
A. A meeting
B. A job position
C. A discussion about story ideas
D. A writing class
Script
M: So, ?
W: Actually, . Would there
be for me to do that here?
M: Well, the truth is that writing is not . We
have a writing department that story ideas and articles. But,
with the writing
project manager to discuss the possibility.
Paraphrasing practice
1. They‟re hiring for other positions 

2. The company is being reorganized 

3. He will resign 

4. I could be transferred there 

5. Receive my paycheck 
6. When is payday? 

7. Give employees bonuses 

8. We owe our success to your dedication 

9. Take a vacation 

10.I don‟t have to work tomorrow 

11.Find out who‟s available for overtime 

12.He is going to be the speaker 

13.I couldn‟t make it to the reception 

Practice
1. How long is the Design Expo this year? 7. What does the man want to know?
A. 2 days A. Whether he will be transferred
B. 3 days B. How Ms. Cole got a new desk
C. 4 days C. What happened to Ms. Cole
D. 5 days D. When the boss will leave for San
2. What does the man say about the expo? Francisco
A. He cannot find the time to attend 8. Where is Ms. Cole now?
B. He was too busy to go last year A. In San Francisco
C. It is good for making business contacts B. At a going-away party
D. It is be held in a different city this year C. In another department
3. What does the woman suggest the D. In the manager‟s office
man do? 9. What is suggested about Ms. Cole?
A. Request more time off work A. Her work allows her to visit San
B. Set up a display at the expo Francisco
C. Not attend the expo this year B. She will visit her old branch often
D. Ask a friend to go along with him C. She is planning a party for her
office
TOEIC FOCUS ---- TOEIC BUILDING-----Smart Home for Toeic

4. Why does the woman want to speak D. Her transfer will be approved the
with Tracy? next day
A. To apply for a job
B. To ask for a discount
C. To request her phone number
D. To suggest hiring more workers
5. What does the woman say she will
do next?
A. Send an e-mail
B. Check the budget
C. Make a phone call
D. Do some extra work
6. What is the woman concerned about?
A. The staff may be overworked
B. The customers will not be satisfied
C. Sales have decreased
D. They will not have enough money
3. General business, finance and budgeting, and corporate development
*** Common situations
1. General business
- Contracts, negotiations, mergers, marketing, sales, warranties, business planning,
conferences, labor relations, etc.
2. Finance and budgeting
- Banking, investments, taxes, accounting, billing, etc.
3. Corporate development
- Research, product development, etc.
*** Typical questions
a, Overview questions
1. The topic / purpose of a conversation
Ex1. What are the speakers mainly discussing?  A proposed budget
Ex2. What is the purpose of the telephone call?  To check on a delivery
2. The speaker’s or speakers’ occupation/ position
Ex: who most likely is the man?  A salesperson
3. The location of a conversation
Ex. Where does this conversation probably take place?  At a manufacturing plant

80
TOEIC FOCUS ---- TOEIC BUILDING-----Smart Home for Toeic

b, Detail questions
4. A specific problem
Ex1. What is the woman‟s problem?  She did not know a deadline was
changed Ex2. What are the speakers concerned about?  The price of a product
5. What to do next
Ex. What will the woman probably to next?  Contact the supply company
6. A specific suggestion
Ex1. What does the woman suggest?  Finding a new supplier
Ex2. What does the man ask the woman to do?  Hold a team meeting / Provide a cost estimate
7. A specific piece of information
Ex. What does the man say about Tokyo expense?  The cost seems high
Example
Q1. What is the man concerned about?
A. The event might not attract enough attendees
B. The new catalog is too expensive to make
C. The catalog may not draw buyers
D. The sales record for the fall season was discouraging
Q2. What does the woman suggest to solve the problem?
A. Running a newspaper advertisement
B. Designing better-looking catalogs
C. Spending more on display space
D. Changing their merchandise inventory
Q3. Why does the man want to hold a company event?
A. To get to know some of their customers
B. To test a strategy they have never tried
C. To thank regular buyers for their loyalty
D. To introduce buyers to the latest styles
Script
M: The latest fall styles are in, but .
W: Me too. Last spring, we spent a fortune on catalogs, but got very little in sales.
.

81
M: That‟s an idea, but .
W: That‟s a that might work.
Paraphrasing practice
1. Place an advertisement in a newspaper 
2. Launch a series of new advertisements 
3. Toys flew off the shelves 
4. When business picks up 
5. Sales are slow 
6. The price was too steep 
7. Business is very good 
8. Be awarded the account 
9. Win the contract 
10. Cut costs by modernizing the assembly lines  _
11. To cover costs 
12. Some stores are less profitable 
13. Postpone replacing some cars and limousines  _

Practice
1. Why is the woman calling? 7. What are the speakers
A. To discuss recent bad weather mainly discussing?
B. To discuss a shipment A. New products
C. To discuss container ships B. Delivery problems
D. To discuss stock supplies C. Marketing strategies
2. What was the cause of the delay? D. Business finances
A. The supplier sent it late 8. What does the man
B. The weather was bad suggest?
C. The shipment was mislabeled A. Recalculating the figures
D. The factory was on strike B. Finding a new delivery company
3. How long were the ships held up C. Reducing the advertising budget
at the port? D. Cutting back on supplies
A. 7 days 9. What will the woman probably
B. 10 days do next?
C. 25 days A. Contact some companies
D. 30 days B. Send an estimate
4. What is being discussed? C. Talk to a department manager
D. Review the sales figures
A. A logistics strategy
B. A design submission
C. A meeting agenda
D. A merger proposal
5. What is causing the delay?
A. The high number of proposals
B. Detailed planning
C. The renovation
D. The winter holidays
6. When is work on the project likely
to begin?
A. At the beginning of November
B. In the middle of December
C. At the end of December
D. In the middle of January

4. Everyday situations
*** Common situations:
1. Traffic, shopping, housing
- Traffic, buying and renting, moving out, moving in, etc.
2. Public facilities
- Restaurants, hospitals, dry-cleaner‟s, post offices, libraries, banks, parking lots, etc.
*** Typical questions:
a, Overview questions
1. The topic/ purpose of a conversation
Ex. What are the speakers mainly discussing?
 What they‟ll do this weekend
2. The speaker’s or speakers’ occupation/ position
Ex. Who most likely is the man?
 A parking attendant
b, Detail questions
1. A specific problem
Ex. What is the problem?
 Tables are not available for the requested date.
2. What to do next
Ex. What will the man probably to next?
 Help the woman locate a product
3. A specific suggestion
Ex. What does the woman ask the man to do?
 Recommend some food / Complete a form
4. A specific piece of information
Ex. What does the man say about his apartment?
 It is near his workplace
5. A specific desire
Ex. What does the man want to do?
 Open a bank account
6. A specific time
Ex. When does the business close?
 At 7p.m
Example:
Q1. Where most likely does the man work?
A. At a university
B. At a printer‟s
C. At an advertising company
D. At a real estate agency
Q2. Where did the woman see the advertisement?
A. In a magazine
B. In a newspaper
C. On a poster
D. On a flyer
Q3. What time does the man suggest the woman meet him?
A. 12 o‟clock
B. 1 o‟clock
C. 2 o‟clock
D. 3 o‟clock
Script
W: Hi, .
M: I‟m sorry, but that apartment has already been taken. However, I do have a one
that‟s close by.
W: Great, could I see it this weekend?
M: Let me see. .
Paraphrasing practice
1. The highway is starting to get crowed. 
2. There isn‟t much traffic 
3. Change buses 
4. It skips that station 
5. It‟s just around the corner from the store 
6. We have a special on oranges 
7. The bag has a slight scratch 
8. The camera lens is chipped 
9. This coat is too small 
10. I want to get a refund 
11. The ingredients could be fresher 
12. The firm is picking up the check 
13. My daughter is in bed with the flu 
Practice
1. Where most likely does the 7. Why does the woman call the
woman suggest going? post office?
A. To a park A. To ask the cost of mailing a
B. To a coffee shop package
C. To a restaurant B. To request a customs form
D. To a sandwich shop C. To inquire about business hours
2. What does the man say about the D. To find out where it is located
Blue Moon? 8. What does the man say
A. It is too far from the office about international
B. It does not deliver packages?
C. It is too expensive A. They cost more to send
D. It is too crowed B. They take longer to deliver
3. What will they probably do next? C. They require a customs form
D. They are sent every Saturday
TOEIC FOCUS ---- TOEIC BUILDING-----Smart Home for Toeic

A. Go to a park 9. According to the man, what can


B. Pick up some coffee the woman do to save time?
C. Return to the office A. Come later in the day
D. Ask a friend to join them B. Fill out the forms in advance
------------------------------------------- C. Send the package by express mail
4. What does the woman want to do? D. Call ahead to make an appointment
A. Buy a blouse -------------------------------------------
B. Pay for a shirt
C. Exchange a skirt
D. Try on a suit
5. What color does the woman want?
A. Black
B. White
C. Red
D. Gray
6. What does the man offer to do?
A. Look in the stockroom
B. Find a smaller size
C. Give her store credit
D. Contact other

5. Travel and free time


*** Common situations
1. Travel
- Airplanes, taxis, buses, ships, ferries, tickets, schedules, station and airport announcements,
car rentals, hotels, reservations, delays and cancellations, etc.
2. Activities in the free time
- Business and informal lunches, banquets, receptions, restaurant reservations, cinema,
theater, music, art, exhibitions, museums, media, etc.
*** Typical questions
a, Overview questions
1. The topic/ purpose of a conservation
Ex. What are the speakers discussing?
 a room reservation/ The man‟s vacation plan/ A play
2. The speaker’s or speakers’ occupation/ position

86
TOEIC FOCUS ---- TOEIC BUILDING-----Smart Home for Toeic

Ex. Who is the woman probably talking to?  a tour guide


3. The location of a conversation
Ex. Where does this conversation probably take place?  in a hotel/ at an airport/ in a museum
b, Detail questions
1. A specific problem
Ex1. What is the woman‟s problem?  she missed a flight
Ex2. What is the man concerned about?  the cost of the tickets
2. What to do next
Ex. What will the man do next?  make a reservation
3. A specific suggestion
Ex. What does the woman suggest?  making the flight reservation sooner
4. A specific time
Ex. When does the woman‟s flight leave?  Tuesday morning
5. A specific location
Ex. Where is the woman‟s final destination?  New York
Example
Q1. What information does the woman provide?
A. Flying advice
B. Ticket prices
C. Carry-on limits
D. Flight schedules
Q2. How many indirect flights are available per day?
A. One
B. Three
C. Five
D. Nine
Q3. What time does the man want to leave?
A. At 9:30 a.m
B. At 11:15 a.m
C. At 12:30 p.m
D. At 5:30 p.m

87
Script
M: Hi, I‟d like to book a flight from San Francisco to Atlanta for tomorrow
W: Well, . There are also flights
available that stop in Salt Lake City to pick up passengers.
.
M: I see. I‟d like to take the earliest one.
W: Let me check. Ok, but we only have . Is that alright?
Paraphrasing practice
1. Make plane reservations 
2. Make sure the airplane reservation is set 
3. The flight doesn‟t leave until nearly midnight 
4. I plan to drive back  _
5. Wait at the gate to board the plane 
6. The flight took off late 
7. Book a room 
8. The shows are all sold out 
9. I think her first book is a lot better 
10. Come back to this museum next month 
Practice
1. Who is the woman probably talking to? 7. Where does this conversation take place?
A. A receptionist A. At a luggage store
B. A flight attendant B. At an airport
C. A bus driver C. At a theater
D. A tour guide D. At a train station
2. Why is the woman unable to come at 8. What does the man suggest the
9 o’clock? woman do?
A. She will be out shopping A. Be ready to board early
B. She will be visiting the zoo B. Purchase a smaller set of bags
C. She will be on another tour C. Store her luggage overhead
D. She will arrive later in the day D. Postpone her trip by a week
3. Where can the woman catch the bus? 9. What is the woman’s problem?
A. In front of her hotel A. She cannot find her set of luggage
B. Down the street B. She has misplaced her reservation
C. At the tour center C. She has to return at an earlier date
D. At the train station D. She cannot take her bag on the plane
TOEIC FOCUS ---- TOEIC BUILDING-----Smart Home for Toeic

4. Where does the conversation


probably take place?
A. At a hotel
B. At an art gallery
C. At a restaurant
D. At a theater
5. What is the woman’s problem?
A. She does not know where to sit
B. She cannot find her friends
C. She needs to locate the ticket counter
D. She lost her reservation number
6. What will happen if the woman is late?
A. She will not be able to buy tickets
B. She may miss the beginning of a show
C. She will be locked out of her hotel room
D. She may lose her reservation
-------------------------------------------------------

PRACTICE TEST

1. Where are the speakers 7. What are the speakers discussing?


probably working? A. A play
A. At an accounting office B. A circus performance
B. At a university C. A music concert
C. At a publishing company D. A new movie
D. At a job placement 8. Why has the woman not yet
2. How many interns will the attended the show?
speakers have this year? A. She is not interest in the show
A. Two B. The performance is too far away
B. Three C. She will be out of town
C. Five D. She does not have much free time
D. Seven 9. How will the woman most likely get
3. What does the woman suggest to Sydney?
about Matt? A. By car
A. He is majoring in accounting B. By train
B. He would be helpful in research C. By plane
C. He is studying linguistics D. By bus
D. He would work well in editing

4. Who most likely is the man? 10. How many days will the man stay in

89
A. A fashion designer Paris?
B. A cashier A. Two
C. A salesperson B. Four
D. An operator C. Five
5. What does the man offer to do for D. Six
the woman? 11. Why does the man need to call
A. Place a special order the airline?
B. Give her a discount A. To select seats for his next flight
C. Connect her to another department B. To ask about baggage allowances
D. Check the stockroom C. To inform of dietary restrictions
6. What will the woman probably do next? D. To confirm the flight schedule
A. Exchange a purchase 12. Who is the man traveling with?
B. Look at other items A. His wife
C. Call back later B. A coworker
D. Look for a smaller size C. His friend
D. A client

13. What problem does the 19. When is the conversation taking place?
woman mention? A. In the morning
A. The internet is not working B. In the afternoon
B. The advertising campaign failed C. In the evening
C. The sales volume has fallen D. On the weekend
D. The staff cannot fill new orders
14. What does the man propose doing 20. What does the man suggest?
to solve the problem? A. Coming back in the evening
A. Establishing a delivery service B. Trying another hotel
B. Offering an online purchasing service C. Filling out a reservation form
C. Advertising in a popular magazine D. Reserving a room for tomorrow
D. Changing the brand name of
their products 21. What does the man say about
15. What will the speakers probably the hotel?
do next? A. It is hosting a convention
A. Get in touch with Galaxy Products B. It is closed for renovations
B. Consult the company‟s director C. It has vacancies for the weekend
C. Send promotional brochures D. It is more modern than the other hotel
D. Call a web design company

16. Who most likely is Grace Park? 22. What is the woman calling about?
A. An office manager A. A late delivery
B. An interior designer B. An unpaid expense
C. A new employee C. A bank statement
D. A secretary D. Travel reservations
23. What department does the woman
17. Why is the woman concerned? work in?
A. The design plans are not finished A. The legal department
B. The office will not be ready on time B. The accounting department
C. The cost of renovation is high C. The travel department
D. There are not enough workers D. The personnel department
18. What does the man say he will do next? 24. What does the offer to do?
A. Look for an available office A. Send the payment
B. Contact Ms. Park B. Contact the bank
C. Speak with the office manager C. Deposit the money
D. Finish the office designs D. Book the ticket

25. Why is the man calling? 28. What day is the staff meeting?
A. To complain about service A. Today
B. To order some items B. Tomorrow
C. To discuss an invoice C. On Monday
D. To report a problem with some equipment D. On Friday
26. What did the company receive 29. Who will be introduced at
on Monday? the meeting?
A. Some printers A. The new office manager
B. Some copiers B. A corporate lawyer
C. Some papers C. A department store representation
D. Some computers D. An insurance salesperson
27. What will the woman probably do next? 30. Why is the woman concerned?
A. Contact another department A. She does not have insurance
B. Offer a refund for defective products B. She may be late for the meeting
C. Take the man‟s order C. Her meeting was cancelled
D. Change the order number D. She cannot work on Monday
TOEIC FOCUS ---- TOEIC BUILDING-----Smart Home for Toeic

PART 4: SHORT TALKS


LESSON 1: TYPES OF QUESTIONS
1. Overview questions
 Questions about the main idea or purpose of a talk
Typical questions
Main idea
What is being announced?
What is the speaker discussing?
What is this report about?
What is being advertised?
Purpose
What is the purpose of this announcement?
What is the purpose of the talk ?
Example
Q. What is the purpose of the message ?
A . To make an appointment
B. To offer a position
C. To cancel an interview
D. To confirm receipt of an application
Script
Hello, Mr Nakamura. This is Eric Parkman, manager of the
for Linderman Enterprises. We received your résumé last
week for the accounting position in our . I was very impressed
with your and . I would like to an
at our office on Friday, April 29 at 10 o‟clock. Please give me a call and let me
know it that time is good for you. Thank you.
 Questions about the speaker and the audience
Typical questions
The speaker
Who most likely is the
speaker? Who is speaking?

92
Who is addressing the audience?
What field does the speaker probably work in?
The audience
Who is the speaker most likely talking to?
Who is this report for?
Who is this advertisement for?
Who is the intended audience for these instructions?
Example
Q. Who is the speaker?
A. A new employee
B. The founder
C. The chairman of the board
D. The personnel director
Script
Staff members of Winton , welcome to the 8th annual Top
Achievers Awards Dinner. When I founded Winton Property Development 13 years ago, I knew
that I alone could not make the company the that it is today. I knew that it
would take people like you help me that goal. The reason why I
this dinner every year is to express my deep to everyone for working so
hard for the continued of this company.
 Questions about the location
Typical questions
Where is this announcement being made?
Where does the announcement take place?
Where are the people?
Where is the talk taking place?
Where is the speaker?
Where is the speech probably being given?
Example
Q. Where is this announcement being made?
A. At a super market
B. At a bookstore
C. At a library
D. At a department store
Script
May I have your , please? This building will be closing in 15 minutes. At this time,
please take all books you intend to to the circulation desk and place those
you will not be borrowing on the re-shelving carts. Those using computers, please note that the
will automatically shut down in 5 minutes. Please save your work to a disk,
your trash folder, and immediately. Because of the public holiday,
this will be closed tomorrow and . We will reopen again at 10:30
Friday morning and will close at 6 that evening. We will our regular operating hours
on Monday the 15th
Practice
1. What is the purpose of the talk? 7. Where is the announcement most
A. To address customer complaints likely being made?
B. To discuss a presentation A. At an airport
C. To celebrate a new contract B. At a hotel
D. To hand out assignments C. At a train station
2. What does the speaker say D. At a bus station
about Confederated 8. What causes the problem?
Enterprises? A. A mechanical malfunction
A. Representatives will meet at the B. Poor weather
company in a couple of days C. An airport closure
B. The company will reviewing their D. A bus accident
factory in a few months 9. What will be provided for the travellers?
C. The company did not meet their A. A free flight
deadline B. Meal vouchers
D. Work for the company must be C. Winter clothing
completed in 4 months D. Hotel rooms
3. What does the speaker ask for?
A. Additional workers
B. A detailed work plan
C. The cancellation of the other projects
D. A review of the contract.

4. Who most likely is the speaker?


A. A landlord
B. A business executive
C. A real estate agent
D. A police officer
5. What does the speaker say about
the office space?
A. It is quite large
B. It is near the train station
C. It is affordable
D. It has a nice view
6. What does the speaker ask Marty to do?
A. Contact the landlord
B. Call her mobile phone
C. Stop by later
D. Arrange an appointment

2. Detail questions
Typical questions
What are employees asked to do on Monday?
What does the speaker suggest the listeners do?
What will the audience probably do next?
What will the tourists do next?
What will happen after the talk?
What will happen on Friday?
What does the speaker say about the traffic?
Example
Q1. What is the celebration for?
A. A merger
B. An anniversary
C. A new CEO
D. A stock offering
Q2. What will William Casper be discussing?
A. Financial strategies
B. New guidelines for employees
C. The history of the company
D. Plans for the future
Script
It is my great to welcome you all to this evening‟s anniversary

. In just over a decade, we have become one of the most reputable names in
the American industry. It is largely because of your that we
have made such tremendous strides. In a few minutes, our chief operating officer, Willis
McGhee, will give a overview of what our company has achieved over the past 10
years. Following his comments, we will be treated to a by our founder
and officer, William Casper, how the company can build its
success in the coming years.
 Question with Why
Typical questions
Why is the speaker calling?
Why is the speaker
apologizing? Example
Q1. Why is the speaker calling?
A. To schedule another interview
B. To offer a job position
C. To make a business proposal
D. To request additional information
Q2. Why does the speaker want to hire someone quickly?
A. Their latest project needs to be edited
B. They want to begin a new project soon.
C. They are facing a publishing deadline
D. They have s shortage of personnel.
Script
Hello, Lucy. This is Darren Norton, with the Hudson Institute for Economic Research. It was a
pleasure meeting you on Monday. The other members of the
and I were quite impressed with both your resume and
. In particular, we found your most recently published works to be very
to our needs. If you are interested we would like you to come back next
Monday for a second interview. Our current will be finished in 15 days and we
would like to have our new team in place as soon as possible because work
on our next project will begin very soon. Please return this call at your earliest
so we can establish a time.
FC
 Question with Who
Typical questions
Who is Laura Thomson?
Who made a phone call?
Who approves product for shipping?
Example
Q. Who is Gloria Lasser?
A. A talk show host
B. An industry executive
C. Apublic health official
D. A popular author
Script
Welcome to Every Opinion That Counts, the town of Oakwood‟s weekly
program, I‟m your host, Charles Tanner. In the studio today, we
have Gloria Lasser, whose new book, “Food Hazards of the modern age” has made quite a splash
since it came out last spring. It has sold nearly _ copies, won the Wiser Prize
for Investigative Reporting and even forced certain facilities to improve
their of hygiene. According to her research, if more companies in the food
business don‟t clean up their plants, the and

of food poisoning will grow dramatically in the next decade. Today,


MsLasser will share some of her ideas about how to further reform the food industry.
 Questions with How:
Typical questions
Methods / means
How can the listener get more information?
How can people request a service?
How can the staff reserve the meeting room?
How does the speaker suggest they handle the problem?
Frequency / Duration / Quantity
How often is this report broadcast?
How long has Dr. William worked for the company?
How many proposals were received?
Example:
Q1. How often is the competition held?
A. Once a year
B. Once every six months
C. Once a month
D. Once every other year
Q2. How can students get partial scholarship?
A. By achieving top marks in high school
B. By ranking the highest in the competition
C. By scoring higher than ninety percent
D. By applying for financial aid.
Script
The Beckman Higher Education Fund is committed to providing
for deserving young people who want to pursue higher
education in science. Each year, the fund holds a competition in which high-achieving high
school students from across the state take a test designed to their ability in
science. All of those who receive a score greater than 90 percent will receive 50 percent off of
their first-year for any program in the state. And, as in
previous years, a full four-year for study at university in the country
will be given to the student with the highest overall score. If you feel that your child is up to the
task, please see me for an after the program today.
 Questions with When/Where/Which
Typical questions
When
When does the library close?
When will Ms. Lane‟s flight arrive?
Where
Where was the suitcase found?
Where will the conference be
held? Which
Which job position needs to be filled?
Example
Q1. Where are the passengers instructed to go?
A. Gate 12-D
B. Gate 13- D
C. Gate 14- D
D. Gate 15- D
Q2. When was the original departure time?
A. 6:30 pm
B. 7:30 pm
C. 8:30 pm
D. 9:30 pm
Script
May I have your attention, please. This is an announcement to all passengers on Eagle Air Flight
327 to Los Angeles. Please be that your departure gate has been .
Instead of Gate 13- D, as indicated on your pass, please use Gate 15-D. In
addition, the departure of your flight has been delayed due to the late arrival of the
from Dallas International Airport, where weather forced a temporary
suspension of flights earlier today. The new departure time will be 8 pm, 30 minutes later than
originally scheduled. As a result, the arrival time in Los Angeles will also be delayed by 30
minutes. Eagle Air would like to take this opportunity to for any inconvenience
caused.
Practice
1. How long is the tour? 7. Why is Springfield Road closed?
A. 15 minutes A. There was a car accident
B. An hour B. Street repairs are in progress
C. Two hours C. There was a power failure
TOEIC FOCUS ---- TOEIC BUILDING-----Smart Home for Toeic

D. A day D. The water line is damaged


2. What does the man say about 8. According to the report, what
the village? will happen tomorrow?
A. It is quite small A. A bridge will be closed
B. It was restored by the government B. A road will reopen
C. It is one hundred years old. C. A traffic report will be
D. It has many gates. broadcast.
3. According to the talk, what is D. Repairs will begin
available in the gift shop? 9. When will the next report occur?
A. Local artwork A. In 20 minutes
B. Traditional souvenirs B. In 28 minutes
C. Pamphlets about the village C. At 2 o‟clock
D. Books about the area. D. Tomorrow morning.

4. Where has the work not


been completed?
A. In the lobby
B. In the lounge
C. In the reception area
D. In the break room.
5. When must employees put away
their possessions
A. On Monday
B. On Tuesday
C. On Wednesday
D. On Thursday.
6. What are the listeners asked to do?
A. Clear out lockers
B. Wear their uniforms
C. Empty their closets
D. Take a break

10
1
TOEIC FOCUS ---- TOEIC BUILDING-----Smart Home for Toeic

PRACTICE TEST
1. Who is the intended audience for 7. Where would you hear this talk?
the announcement? A. At a historical site
A. Produce deliverers B. In a museum
B. Library patrons C. At a zoo
C. Shop employees D. In an art gallery
D. Store customers. 8. What does the speaker ask the
2. What are the listeners asked to do? visitors to do?
A. Head to checkout counters A. Refrain from talking
B. Visit the produce section B. Stay away from the cages
C. Return in the morning C. Buy souvenirs at the gift shop
D. Make their orders D. Watch a film after the tour
3. What time does the business close? 9. Where will the tour end?
A. 8 pm A. In the children‟s area
B. 9 pm B. At the activity room
C. 10 pm C. In the gift shop
D. 11pm D. At the cafeteria

4. Who is this report probably for? 10. Who is listening to the talk?
A. Construction workers A. Elementary schoolchildren
B. Athletes B. Neighborhood residents
C. Radio listeners C. Members of community play
D. Drivers D. Workers at a construction site
5. According to the speaker, what will 11. What is the talk mainly about?
be needed this morning? A. Ways to save on construction costs
A. An umbrella B. Recreational activities for local
B. A raincoat residents
C. Warm clothing C. How to improve traffic conditions
D. A hat D. The best location to build a playground
6. According to the speaker, how will 12. Why is the Portman Road site
the weather be different tomorrow? not being considered?
A. It will be cold A. The land is government property
B. It will be rain B. The area is expensive
C. It will be sunny C. There is too much traffic
D. It will snow D. It is near a large factory

13. What is the speaker calling about 16. For whom was this
A. An order for office furniture information recorded
B. A time for delivery A. Flight attendants

102
C. A location for the company B. Airport employees
D. A room reservation C. Travel agents
14. What does the caller need D. Airline passengers
more information about? 17. Why would a caller press zero?
A. A style of desk A. To make a reservation
B. A type of cabinet B. To check in an arrival time
C. The floor number C. To speak with an agent
D. The placement of thee furniture D. To confirm a departure time
15. What does the caller ask Ms Klein 18. What must callers do to get
to do? information on departing
A. E-mail the specifications flights
B. Send in request A. Provide flight information
C. Sign for the delivery B. Confirm arrival times
D. Return the call C. Book a flight ticket
D. Contact the travel agent.
LESSON 2: TYPES OF TALK
1. Announcements
*** Common situations
1. Announcements given in offices
2. Announcements made on airplanes or in airports
3. Announcements given in stores, at shopping events, and in other situations
*** Typical questions
a, Overview questions
Ex 1) What is being announced? The new company policy
Ex 2) What is the purpose of the announcement?  To report a schedule change
Ex 3) Who is the announcement for?  Airplane passengers
Ex 4) Where is this announcement being made?  At a grocery store
b, Detail questions
Ex 1) What are the listeners asked to do?
 Prepare to leave the office
Ex 2) What change will be introduced next week?
 The store will stay open for longer hours.
Ex 3) When is the museum open on
Mondays?
 At 10 a.m
Example:
Q1. Where is this announcement probably being made?
A. In a travel agency
B. In a train station
C. In an airport
D. In an office
Q2. What should Cindy Clive do?
A. Transfer to her connecting flight
B. Board the airplane immediately
C. Report to the information desk
D. Go to the baggage claim
Q3. What is the courtesy phone near?
A. The departure lounge
B. Terminal 2
C. The Milennium Airlines desk
D. Gate 15 – G
Script
Your attention, please, Milennium Airlines is paging a on Flight 78 to Chicago.
Mrs. Clive, please come as soon as possible to the Milennium Arilines
, located in Terminal 2. , you may
one of our blue courtesy phones located directly beside the to the
, to be connected to a staff member. We have important
concerning your flight to Orlando from Chicago. It is
that you talk to us to obtain this information before your , which is
leaving in from Gate 15 – G. Again, Mrs. Cindy Clive, passenger to Chicago,
please immediately the Milennium Airlines information counter.
Practice
1. When will inspection take place?
A. Tomorrow
B. On Wednesday
C. On Friday
D. Next Monday
2. What will happen if they fail the inspection?
A. Their facility will close for renovation
B. They will have to pay a penalty
C. A second inspection will occur next month
D. Staff who fail to meet standards will be laid off
3. What will managers do to prepare for the inspection?
A. Consult with health inspectors
B. Clean all workstations
C. Go to every workstations
D. Increase sanitation measures
4. Who is the intended audience for the announcement?
A. Environmentalists
B. Restaurant customers
C. Exhibition organizers
D. Tourists
5. What does the speaker say about the Arctic exhibit?
A. It is open for two hours
B. They will visit it in the morning
C. It is closed for renovation
D. They will take pictures of it.
6. What does the speaker recommend?
A. Wearing warm clothing
B. Putting their cameras away
C. Purchasing refreshments after the tour
D. Traveling abroad
7. What is mentioned about the business?
A. It is closing
B. It is not very large
C. It is owned by a family
D. It has recently opened
8. How can customers get a free delivery?
A. By paying with a special coupon
B. By purchasing an item above a certain amount
C. By ordering online
D. By using the store membership card
9. According to the announcement, what has been changed?
A. Store locations
B. Business hours
C. Building owners
D. Customers service
2. Speeches & Talks
*** Common situations
1. Speeches or talks given at social events
2. Speeches or talks given at business meetings
*** Typical questions
a, Overview question
Ex 1) What is the main purpose of this speech?  To thank supporters
Ex 2) Where is the speech taking place?  At an awards ceremony
Ex 3) Who is the speech probably for?  Museum visitors
b, Detail questions
Ex 1) Who is Sandra Clark?  A successful manager
Ex 2) What does the speaker say about his regional office?  They increased sales
Ex 3) What will probably happen next?  A guest will speak.
Example
Q1. What is the purpose of the meeting?
A. To sell software
B. To address complaints
C. To provide training
D. To master typing
Q2. What new features are mentioned?
A. Spell-check functions
B. Language options
C. Spreadsheet functions
D. Sound functions
Q3. When is the session taking place?
A. During break time
B. During working hours
C. During lunch hours
D. After work
Script
This morning‟s is about using WordMine, the latest word processing
software from Lexicon. Although WordMine has all of the old , there are some
new that you will be learning for the first time. It also has added
not commonly found in word processing programs. Once you master these,
you will find WordMine a tool for making . We know that
this is taking time out of your , so we will try to finish as quickly
as possible. If you have any questions, feel free to ask at any time. Let‟s start by looking at the
in the in front of you.
Practice
1. Why is the audience gathered?
A. To exhibit designs for a building
B. To raise money for a project
C. To celebrate the opening of the hall
D. To ask for additional donations
2. What has Mr.Collins done?
A. Designed a building
B. Given money to the project
C. Donated land
D. Offered to perform
3. What will probably happen next?
A. A closing ceremony
B. A musical performance
C. A welcome dinner
D. A city meeting
4. What is the main purpose of this talk?
A. To introduce a speaker
B. To inform listeners of a new company
C. To arrange a seminar
D. To promote a university
5. Who is Dr. Alana Miles?
A. A publisher
B. An entertainer
C. A university professor
D. A medical doctor
6. What did Dr. Miles do 5 years ago?
A. She studied public relations
B. She arranged a seminar
C. She promoted a university
D. She had a book published
7. Where is the speaker?
A. At a sports event
B. At a restaurant
C. At a sales meeting
D. At a park
8. What is the speaker talking about?
A. An employee picnic
B. A special meal
C. A successful marketing campaign
D. A sales presentation
9. Who is Julie Kavner?
A. An event planner
B. A successful manager
C. A new chef
D. A salesperson

3. Guided tours
*** Common situations
1. Talks at public buildings
2. Others: on a picnic/bus/sightseeing tour
*** Typical questions:
a, Overview questions
Ex. 1) Who most likely is the speaker?  A tour
guide Ex. 2) Where does the talk take place?  At a
factory
Ex. 3) What is the main purpose of the talk? To describe a tour
b, Detail questions
Ex. 1) What does the speaker say about the house?  It will become a hotel
Ex. 2) What will happen at 5?  A tour will end
Ex. 3) What does the speaker encourage people to do?  Drink water
Ex. 4) Where will the people probably go next?  To a packaging room
Example:
Q1: Where is this announcement being made?
A. At a hotel
B. On a cruse shop
C. At a health club
D. On an island
Q2: What activity does the speaker mention for Saluki?
A. Fishing in a lagoon
B. Touring nearby museums
C. Eating varied cuisine
D. Scuba diving
Q3. What time will they depart for Hacienda?
A. 2 p.m
B. 3 p.m
C. 4 p.m
D. 5 p.m
Script
Welcome aboard the Saint Meridian. We‟ll be in just a few
minutes. About after we _ the New London harbor, we will arrive at the
beautiful of Saluki. We‟ll spend about 4 hours just off its coast,
which will give you the chance to do some . Don‟t miss the
to see the in the island‟s lagoon, as many of these
aren‟t found anywhere else in the world. Then, at 3 p.m, we will be leaving for the
island of Hacienda, where you will have a wide of dinner options to choose from.
We‟re you‟re with us. Enjoy your voyage!
Practice
1. Where most likely is the speaker?
(A) At a company workshop
(B) At an electronics convention
(C) At a home appliances store
(D) At a reality exposition
2. What new feature is mentioned by the speaker?
(A) Workshops will be conducted
(B) A free dinner is provided
(C) Exhibition areas are larger
(D) Rooms have internet access
3. Why has the location of the dinner been changed?
(A) The room sixe is inadequate
(B) The facility lacks equipment
(C) The previous workshop finishes late
(D) The rent is too high
4. Who is making the announcement?
(A) A history expert
(B) A town official
(C) A tour guide
(D) A bus driver
5. What does the speaker suggest?
(A) Going to a particular restaurant
(B) Visiting historical sites
(C) Boarding the bus early
(D) Not taking too long in the museums
6. What is mentioned about the Old Court House?
(A) What is was used for
(B) Why it was built
(C) How old it is
(D) Who built it
7. What is the speaker introducing?
(A) A robotic demonstration
(B) A film about the ocean
(C) A cruise tour
(D) An award presentation
8. Who is Sharon flock?
(A) A sea explorer
(B) A writer
(C) An actress
(D) A scientist
9. How long will the program be?
(A) 30 minutes
(B) 60 minutes
(C) 80 minutes
(D) 120 minutes
4. Advertisements
*** Common situations :
1. Advertisements for product
2. Advertisements for services
*** Typical questions
a, Overview questions:
Ex 1) What is the advertisement for?  A furniture store
Ex 2) Who most likely is the speaker?  A medical doctor
Ex 3) Who is this advertisement for?  New drivers
b, Detail questions:
Ex 1) What does the speaker say about the product?  It is easy to use
Ex 2) What day does the special offer end?  on Sunday
Ex 3) How can customers get more information?  by making a phone call
Example:
Q1: What is being advertisements?
A: Home appliances
B: A service agreement
C: A 24- hour security system
D: Personal consultations
Q2: What does the speaker mention about technicians?
A: They use the newest equipment
B: They work at a factory
C: They are skilled
D: They will come on the next day
Q3: How long does the offer last?
A: Until the end of year
B: For today only
C: Until the end of this week
D: For 24 days
Script
Count on us to help you when a breaks down. We are here to
solve all your problems. Home Helper, Inc. is offering a one-year service for all
your home for only a dollar a day. Yes, for just one dollar a day, you will
receive our 24-hour service for all of your heaters, ,
, or any other home appliances, Our formally trained and
technicians will provide all the necessary parts and labor to your
products to like-new condition, quickly and . This offer
will only last until this Sunday, so don‟t wait till it‟s too late. Give us a call today at 555-7295
Practice
1. What does the advertised business sell?
A. Plants and trees
B. Clothes
C. Coffee
D. Sporting goods
2. When will the lessons begin?
A. This Monday
B. Next week
C. At the end of this month
D. At the beginning of next month
3. What is located across from the advertised business?
A. A café
B. A shoe store
C. A golf course
D. A school
4. What is this advertisement for?
A. A new restaurant
B. A lunch menu
C. A beverage
D. A health club
5. What is convenient about the product?
A. It is easy to carry
B. It can be delivered
C. It can be heated in an oven
D. It is smaller than a salad
6. What does the advertisement say is special about the product?
A. Its old tradition
B. Its nutritional value
C. Its low cost
D. Its availability
7. What is being advertised?
A. An art exhibition
B. A sports event
C. A radio show
D. A theater
8. Who is Julia Sugarbaker?
A. A radio host
B. A web designer
C. A restaurant manager
D. An event planner
9. How can listeners learn about program topics?
A. By making a phone call
B. By sending an e-mail
C. By visiting the website
D. By listening to later announcement
5. Broadcasts
*** Common situations
1. Broadcasts on traffic situations
2. Weather forecasts
3. Ordinary broadcasts
*** Typical question
a, Overview questions
Ex 1) What is the broadcast mainly about?  Traffic conditions
Ex 2) Who most likely is the speaker?  A radio broadcaster
Ex 3) Who is this broadcast probably for?  Car drivers
b, Detail questions
Ex 1) What does the speaker suggest listeners do?  Taking an alternate route
Ex 2) What will be broadcast next?  A news update
Ex 3) How will the weather be tomorrow?  It will be much colder.
Ex 4) What is causing a traffic delay?  A bridge has been closed.
Ex 5) When will the next weather broadcast take place?  In an hour
Example
1. What does the broadcast mainly concern?
A. A new expressway
B. Construction information
C. A traffic report
D. Regional news
2. According to the speaker, what is the causing the problem on Highway 6?
A. Bridge repairs
B. Street construction
C. Poor weather conditions
D. An automobile accident
3. What will listeners probably hear next?
A. A commercial
B. A weather forecast
C. A talk show
D. A sports report
Script
You‟re listening to the K-69 morning . Motorists heading to the
area should be aware that Bay Harbor Bridge is closing this week for
repairs. Commuters should use Victoria Bridge or McDonald Brige to enter the
. Traffic is heavy on Capital Road leading up to Victoria Bridge, so expect
some . Those headed out of the city are advised that Highway 6 is backed due to a
. Traffic is on the Trans-City
Expressway and Highway 13. This is Brendon Doherly repairing for radio K-649. Our next
report will be in 20 minutes. Stay tuned for local weather, right after our
.
Practice

1. What will temperature be tonight?


A. Below 0
B. About 17 degrees
C. Between 0 and 10 degrees
D. Over 20 degrees
2. What will the weather be like in the weekend?
A. Sunny
B. Rainy
C. Cold
D. Overcast
3. What will listeners hear next?
A review of the top news stories
B. The long-term weather forecast
C. Commercial break
D. Current sports news
4. What is causing the problem?
A. Hot weather conditions
B. Watering lawns
C. A decrease in temperature
D. Local farmers
5. What does the speaker suggest?
A. Voting in a local election
B. Limiting water usage
C. Planting a garden
D. Making reservations
6. When are temperatures expected to rise?
A. On Monday
B. On Friday
C. On Saturday
D. On Sunday
7. What is the subject of today’s program?
A. A new film
B. The movie industry
C. Local elections
D. Popular directors
8. What will happen after interview?
A. A new announcer will be introduced.
B. Election results will be announced
C. Listeners will call the radio station with questions.
D. The host will introduce a guest host.
9. Who will be the host for tomorrow’s program?
A. Stephanie Spielburg
B. An independent film director
C. The speaker‟s co-host
D. A relative of the regular host.
6. Report
*** Common situations
1. Business report
2. News reports
3. Public service bulletins
*** Typical questions
a, Overview questions
Ex 1) What is the report about?  Company acquisition
Ex 2) Who most likely is the speaker?  A news
reporter
b, Detail questions
Ex 1) What does the speaker recommend listeners do?  Use public transportation
Ex 2) What do some people expect will happen?  Negotiation will be conducted
Ex 3) Who is Jennifer Marlow?  A scientific researcher
Example

1. What is the news report about?


A. Exercise and weight loss America
B. Factors increasing risks of heart disease
C. The findings of new heath studies
D. Walking as a new exercise trend
2. According to the report, what has increased in America?
A. The number of overweight Americans
B. The percentage of Americans who exercise daily
C. Visits to hospitals for health problems
D. Prescriptions for patients with heart disease.
3. Who is Will Pearlman?
A. A hospital researcher
B. A university administrator
C. A fitness trainer
D. A heart surgeon
Script
A new study by University Hospital, just out today, reveals that the number of health-conscious
Americans has significantly in the past 5 years. The number of American adults
who exercise for at least a day has increased by over , compared to 5
years ago. In another study, the hospital noted that there has been a in
the number of patients coming to the hospital for treatment for heart . The
Dr. Will Pearlman, noted that adults can significantly reduce the risk
of just by doing a few light every day. This could
involve gently, jogging to work, stairs instead of taking
or even just taking a stroll after meals.
Practice

1. What is the report about?


A. The company‟s financial condition
B. The shutdown of a factory
C. The introduction of a new car model
D. The state of the local economy
2. How many jobs will be lost?
A. 250
B. 350
C. 450
D. 550
3. What will company be offering?
A. Job training
B. Job transfers
C. Financial compensation
D. Overseas work
4. What is the report about?
A. A financial crisis
B. A traffic delay
C. A subway line opening
D. A local suburb
5. When did the project begin?
A. A year ago
B. 2 years ago
C. 4 years ago
D. 5 years ago
6. What delayed the project?
A. A protest by local residents
B. A malfunction with machinery
C. A shortage of travelers
D. A lack of money
7. What is the report about?
A. A newly elected governor
B. A new ferry service
C. A plan to build a bridge
D. A traffic problem
8. How frequently does the Channellink run?
A. Every 15 minutes
B. Every 20 minutes
C. Every 30 minutes
D. Every hour
9. What did the city say about the ChannelLink?
A. It will make commuting by car less attractive.
B. It will require extra funds to operate
C. It will require the construction of a bridge.
D. It will save money for the city
7. Introductions
*** Common situations
1. Introduction to staff member
2. Introductions to others
*** Typical question
a, Overview questions
Ex 1) where is this talk probably taking place?  In an auditorium
Ex 2) What is the purpose of the talk?  To introduce the staff member
b, Detail questions
Ex 1) Who is Steven Garcia?  An energy researcher
Ex 2) What is the topic of Ms. Watson‟s talk?  Solar
energy Ex 3) What was Jerry‟s previous job?  A house
cleaner
Ex 4) What will Ms. Laura probably do next?  Give a speech
Ex 5) What is the audience asked to do?  Write down questions
Example
1. What news does the speaker talk about?
A. An employee‟s transfer to another division
B. A new branch in South America
C. A party to celebrate a contract
D. The retirement of an employee
2. What does the speaker say about Grace?
A. She was not liked by colleagues.
B. She will Join another company
C. She had a distinguished career.
D. She will travel to South America.
3. What does the speaker invite the audience to do?
A. Come to a party
B. Send Grace a card
C. Attend a meeting
D. Talk to Grace
Script
I would like to announce that Grace Melendez. Our long- time representative in South America,
at the of the month. I have worked with Grace for many years, and
have found her both a wonderful and a pleasure to _ with
outside of the office. Her patience and shrewd business won her praise and
from clients and coworkers alike. As our in six
Latin American countries for the last 14 years. She has developed a stellar for
sales. The company‟s into South America is due to largely to her efforts. On
February 3, the company will sponsor for Grace at the LandmarkHotel, to which
you all are invited.
Practice
1. What is the purpose of this talk?
A. To announce a promotion
B. To request something from a branch office
C. To introduce a new employee
D. To ask staff to on a project
2. What is Robbie’s job?
A. A language instructor
B. A visual designer
C. A teacher trainer
D. A Consultant
3. What is mentioned about Robbie?
A. He will start working full-time in 3 months
B. He is fluent in a foreign language.
C. He studied in Germany for a long time.
D. His previous job was in Frankfurt.
4. Who is Alice Collins?
A. A professor
B. A critic
C. A talk show host
D. A write
5. What does the speaker find surprising about Ms. Collins’s success?
A. His first efforts were unpopular.
B. She has little experience
C. Most newspapers disliked her work.
D. She has never taught before.
6. Why is Ms. Collins visiting a University?
A. To speak with student
B. To advertise new work
C. To review a new publication
D. To meet with fans
7. Who is Greg Alberts?
A. A manufacturer
B. An engineer
C. A chemist
D. A pharmacist
8. What is the topic of the session?
A. Building renovation
B. Insurance fraud
C. Occupational safety
D. Equipment updates
9. What does the speaker ask participants to do?
A. Look over some document
B. Attend another session on 3 hours
C. Be punctual for tomorrow‟s session
D. Choose a preferred activity
8. Telephone messages
*** Common situations
A. Record messages by callers
B. Record messages by telephone owners
C. Record messages from public services
*** Typical questions
a, Overview questions
Ex 1) What is the main purpose of the call?  To postpone the appointment
Ex 2) For whom is this message intended?  Job applicants
Ex 3) What type of facility does the messages give information about?  A library
b, Detail questions
Ex 1) Who does the woman work for?  A car repair store
Ex 2) What will happen on December 10?  A presentation will be made
Ex 3) What does the woman ask Mr. Dean to do?  Contact her
Example

1. Why is Mr. Jones making a phone call?


A. To reschedule an appointment
B. To make a business proposal
C. To request information
D. To invite Mrs. Stern to New Orleans
2. Where does Mr. Jones want to meet Mrs. Stern?
A. At his meeting room
B. In the new Orleans headquarters
C. At Alliance Medical
D. At Mrs. Stern‟s office
3. What does Mr. Jones ask Mrs. Tern to do?
A. Send a fax
B. Accept a apology
C. Call him back
D. Schedule a conference
Script
Hello, Mrs. Stern. This is Terry Jones from Alliance medical calling. I am afraid I am going to
have to our meeting on April 11. As you know, I‟ve had to come to New
Orleans to meet with clients. Unfortunately, something has _ and it turns out that
I am going to have to stay here for . However, I would like to suggest
that we meet in on April 23rd by which time I will have returned to
Cincinnati. Please get back to me to let me know if this fits into your . Thanks.
Practice
1. What type of the business has the caller reached?
A. A hospital
B. A hotel
C. A Spa
D. A health club
2. What will begin on July 7?
A. Class registration
B. Swimming classes
C. Repair work
D. Sports competition
3. What information is available at the facility’s website?
A. Revised class schedules
B. Registration form
C. Operating hours
D. Membership requirements
4. What is the purpose of the call?
A. To schedule an interview
B. To make a flight reservation
C. To inquire about a job posting
D. To ask about an investment
5. What is Ms. Botwin’s occupation?
A. An investment banker
B. An account executive
C. A public relations director
D. A travel agent
6. Where will Mr. Botwin be next week?
A. In Shanghai
B. In Seoul
C. In Hong Kong
D. In Osaka
7. What is the main purpose of the messages?
A. To apologize for the wrong shipment
B. To change a delivery date
C. To offer a special deal
D. To confirm a delivery time
8. What is the problem?
A. A salesperson was out of stock
B. The store was out of stock
C. The item need to be rush- ordered.
D. No one was at home to receive the delivery.
9. What does the woman want Mr. Wilson to do?
A. Go to the store
B. Check the delivery address
C. Pay for rush delivery
D. Return all call.
9. Instructions & Lectures
*** Common situations
1. Instructions on work procedures
2. Instructions on use of some public facilities
3. Specialists talks or lectures
***Typical questions
a, Overview questions
Ex 1) What are the instructions about?  Flexible working hours
Ex 2) What is the main purpose of talk?  To explain a new pay system
Ex 3) In what field does the speaker work?  Finance
Ex 4) Who is the intended audience for these instructions?  Sales employees
b, Detail questions
Ex 1) What are employees instructed to do before taking leaves?  Get approval from managers
Ex 2) Why would an employee contact Mr. Thormas?  To request some instructions
Example
Q1. What is the speaker doing?
(A) Explaining how to renew a driver‟s license
(B) Describing the different types of forms
(C) Instructing listeners about obtaining a driver‟s license by mail
(D) Informing patrons about the business hours of the
office Q2.What will the listeners most likely do next?
(A) Enroll in a driving class
(B) Complete a form
(C) Take a driving test
(D) Pay for a license renewal
Q3. Where can applicants get their pictures taken?
(A) At window 1
(B) At window 6
(C) At window 13
(D) At window 14
Script
Those who are here to their , please
listen carefully to the following . The first thing to do is to
the form at Window 1. You should
complete 1 through 10 and through . However, at this point, do not
, as that needs to be by one of
. Bring the form with your current ‟s
to Window , where you will have your picture taken and sign the
. If necessary, move on to Window to be issued a
license to use until your now license arrives in the mail.
Practice
1. What is the main purpose of the talk?
(A) To discuss new security policies
(B) To describe a new scheduling system
(C) To explain office e-mail usage
(D) To provide project assignments
2. What do employees need to have?
(A) A list of e-mail addresses
(B) A password
(C) A security pass
(D) An identity card
3. Why would an employee contact Mr. Pompino?
(A) To request a password
(B) To report computer problems
(C) To ask for an instructional manual
(D) To open an office e-mail account.
4. Who most likely is the intended audience for the instructions?
(A) Travel agents
(B) Trade fair representatives
(C) Sales staff
(D) Accountants
5. What are employees instructed to do after their trip?
(A) Write a sales report
(B) Call the accounting office
(C) Submit receipts
(D) Fill out a form
6. What do employees on trips of five or more days receive?
(A) Overtime play
(B) Laundry
(C) Rental car
(D) Trade fair tickets
7. What is the primary purpose of this talk?
(A) To recommend a certain investment
(B) To provide advice about choosing a broker
(C) To give tips on creating an investment website
(D) To warn about rising internet crime
8. Why does the speaker mention friends and acquaintances?
(A) They may provide you with good advice
(B) They might be able to manage your investments.
(C) They may offer to do the research for you
(D) They might lend you investment funds
9. What does the speaker tell investors to do?
(A) Log in at a certain time
(B) Test the loading speed of the firm‟s website
(C) Have any technical problems fixed
(D) Report faulty links to the webmaster
Practice 1
1. What is the purpose of the talk 4. Who most likely is the speaker?
(A) To announce a decision to move (A) An airport employee
(B) To advertise a new product (B) A flight attendant
(C) To describe the new offices (C) An airplane pilot
(D) To explain the reason for closure (D) A member of a cleaning crew
2. What does the speaker say about 5. What is the problem?
the building on Maple Street? (A) Flight have been delayed
(A) The rent is not high (B) The runways are covered in snow.
(B) Utilities are included. (C) There is a rainstorm.
(C) It is only one story (D) The domestic terminal is closed
(D) It is newly constructed 6. When is the problem expected to
3. What does the speaker plan to do be resolved?
on Friday? (A) In the morning
(A) Show photographs of the location (B) 8 p.m
(B) Sign a rental agreement (C) 10 p.m
(C) Make a conference room (D) Tomorrow
(D) Enlarge the break room

7. According to the report, what caused 10. Who is the speaker?


delays in the morning? (A) A store manager
(A) A bridge construction (B) A salesperson
(B)Poor weather conditions (C) A director of purchasing
(C) A highway closure (D) A store customer
(D) A traffic accident 11. What is the problem?
8. What advice does the speaker give? (A) The wrong order was sent
(A) Use the ferry (B) Some merchandise was defective.
TOEIC FOCUS ---- TOEIC BUILDING-----Smart Home for Toeic

(B) Go over the bridge (C) Too many items were sent.
(C) Take a different route (D) A shipment did not arrive on time
(D) Drive slowly 12. What does the speaker request?
9. According to the speaker, what will (A) An additional shipment.
happen this weekend? (B) A refund.
(A) The construction will begin on a bridge. (C) A discount
(B) The roads will be cleared. (D) An exchange of goods
(C) The highway will be closed.
(D) The weather will be cold.

13. Where is the man calling from? 16. How can auditions be arranged?
(A) An apartment (A) By filling out an application
(B) An office (B) By writing a letter
(C) A train station (C) By calling the theater
(D) A hotel room (D) By having an interview
14. What is not working properly? 17. Who will select the winners of the contest?
(A) A heating system (A) Professional actors
(B) An air conditioner (B) Audience members
(C) A light switch (C) University students
(D) A train (D) A parents‟ group
15. When does the man plan to leave? 18. What prize is being offered?
(A) Immediately (A)An opportunity to perform live
(B) In the morning (B) 400 dollars in cash
(C) At 3 o‟clock (C) Admission to Ossian University
(D) At 5 o‟clock (D) A paid position at the theater

19. What type of business is being 22. What is the speaker’s job?
advertised? (A) A travel agent
(A) An airline (B) A moving company representative
(B) A restaurant (C) An accountant
(C) A hotel (D) A tour guide
(D) A tour agency 23. What does the man need to submit?
20. According to the speaker, what (A) A form
advantage does the business provide? (B) An application
(A) Internet reservations (C) A plane ticket
(B) Mary worldwide destinations (D) A receipt
(C) Computer access

128
(D) Live entertainment 24. What problem does the speaker mention?
21. According to the speaker, what is (A) The file has not been received
special about the company staff? (B) The shipment has not arrived
(A) They are professional cooks (C) The shipment has not arrived
(B) They are hospitality experts. (D) The payment for excess baggage cannot
(C) They have medical training certificates be made
(D) They have business experience

25. How long will the sale last? 28. What is the purpose of the talk?
(A) A day (A) To announce an exhibit
(B) A week (B) To introduce a new colleague
(C) A weekend (C) To gather ideas for a conference
(D) A month (D) To take nominations for an award
26. What can be bought in the store at 29. What will happen in march?
a discounted price? (A) A different art director will work for the
(A) Refrigerators journal
(B) Couches (B) The society will begin an expansion
(C) Computers project.
(D) Sound equipment (C) Several staff members will be hired
27. Who can receive a discount coupon? (D) A new publication will be launched
(A) All shoppers over the weekend 30. What is Alice Evans’s job?
(B) People purchasing MP3 players (A) A vice president
(C) The store‟s first 10 customers (B) An art director
(D) The winner of the coupon (C) An editor
(D) A staff writer

Practice 2
1. Where most likely does the speaker work? 4. Where is the announcement being made?
(A) At a computer repair shop (A) At a zoo
(B) At an electronics store (B) At a shopping mail
(C) At a restaurant (C) At a musical performance
(D) At a service station (D) At the city hall
2. What time of day was the 5. What is offered for free with the
message recorded? purchase of a CD?
(A) In the morning (A) A T-shirt
(B) At noon (B) A poster
(C) In the afternoon (C) A coupon
(D) At night (D) A calendar
3. What does the speaker suggest? 6. How will the money from the sale be used?
(A) Buying a new computer (A) To support animal protection organizations
(B) Meeting for lunch (B) To pay for the concert
(C) Calling the store (C) To refund the customers
(D) installing some new software (D) To protect local neighborhoods

7. How long will the session for the day last? 10. What is the purpose of the message?
(A) 3 hours (A) To ask for directions to a hotel
(B) 4 hours (B) To inquire about a job
(C) 5 hours (C) To make a reservation at a restaurant
(D) 6 hours (D) To request a phone number
8. What will the presenter do first? 11. Where does Mr. Garcia probably work?
(A) Answer questions (A) At a cooking school
(B) Show a video (B) At a bakery
(C) Deliver a report (C) At a hotel restaurant
(D) Introduce the president (D) At an employment center
9. What should the listeners do if they 12. Why does the speaker want Mr. Garcia to
have questions? return her call?
(A) Ask during the break (A) To get a mailing address
(B) Interrupt the speaker (B) To ask about the menu
(C) Wait until the afternoon (C) To request a résumé
(D) Submit questions in writing (D) To find out if the job is still available

13. Where does this announcement 16. Who most likely is the speaker?
most likely take place? (A) A government official
(A) At a theater performance (B) A spokesperson for water utilities
(B) At a music concert (C) A weather forecaster
(C) At a sports event (D) A news reporter
(D) At a photography exhitition
14. What is the main purpose of 17. Who most likely is the speaker?
this annoucement? (A) The rainy season will begin
(A) To inform the audience of rules (B) Temperatures will stay high
(B) To tell about upcoming events (C) Water reserves will increase
(C) To introduce a musical group (D) The weather will improve
(D) To welcome a special guest 18. What can be found on the website
15. According to the announcement, what the speaker mentions?
are available in the lobby? (A) The weather forecast
(A) Events schedules (B) A list of ways to conserve water
(B) Coffee machines (C) A monthly water usage report
(C) Concert tickets (D) Contact information for government
(D) Smoking areas officials

19. Who is the speaker most 22. Who is the speaker?


likely addressing? (A) A bus driver
(A) Film students (B) A professor
(B) Merchandise manufactures (C) A nature expert
(C) Movie critics (D) A tour guide
(D) Cinema audiences 23. What is the purpose of
20. What will the listeners do first? this announcement?
(A) Watch the company‟s new film (A) To indicate a change in lunch plans
(B) Discuss attributes of possible (B) To announce a delayed ruturn time
marketing tactics (C) To advertise a nature tour
(C) Meet senior management for coffee (D) To give directions to the national seashore
(D) Have a meeting of their top sales staff 24. What will the group do at 1.00?
21. Where will the discussion be held? (A) Start the excursion
(A) Upstairs (B) Return to the station
(B) In the next room (C) Have a meal
(C) Downstairs (D) Observe wild cranes
(D) In the screening room

25. Where is this speech probably 28. What is this informationg about?
being given? (A) A phone network failure
(A) At a conference (B) A weather forecast
(B) At a awards ceremony (C) A voicemail system
(C) At a company meeting (D) Customer service
(D) At a convention 29. By when is the work expected to
26. Who is Tony Albright? be completed?
(A) An environmentalist (A) In the evening
(B) An architect (B) Tomorrow morning
(C) An engineer (C) Tomorrow afternoon
(D) A furniture designer (D) In an hour
27. What is described as being 30. How often will the recorded information
energy efficient? be updated?
(A) Natural lighting (A) Every hour
(B) Air conditioning systems (B) Every two hours
(C) Office buildings (C) Every day
(D) Private homes (D) Every week
TOEIC FOCUS ---- TOEIC BUILDING-----Smart Home for Toeic

132
TOEIC
READING
PHẦN 1: ĐỘNG TỪ
LESSON 01
A/ PHÂN LOẠI ĐỘNG TỪ
- Linking Verb

- Nội động từ

- Ngoại động từ

B/ THỰC HÀNH
1. The consulting firm is that the new advertising campaign will
significantly increase market share.
A. Optimist B. Optimistic C. Optimism D. Optimistically
2. Master‟s Choice Beverages provides products to meet the
changing tastes of its customers.
A. Innovator B. Innovative C. Innovate D. Innovations
3. All faculty members are to attend the leadership conference
scheduled at the end of the month.
A. Encourage B. Encouraging C. Encouragement D. Encouraged
4. The flooring materials arrived this afternoon and will be by the
maintenance crew tomorrow morning.
A. Installed B. Installing C. Installment D. Installation
5. The report is thorough and will enable the product development team to
understand the recent upturn in the electronics industry.
A. Extreme B. Extremity C. Extremely D. More extreme
6. All audience members should be of others when watching the
performance in the auditorium.
A. Considerable B. Considering C. Considerate D. Consideration
7. The Royal Jewel Corporation significant revenue increases due to its
recent expansion of its manufacturing facilities in Southeast Asia.
A. Anticipation B. Anticipate C. Anticipates D. Is anticipated
8. The technical team has assured that the new computer system will be
functional by the end of the month, when annual reports are due.
A. Complete B. Completing C. Completely D. Completion
9. Passengers departing from Gate 29 should present boarding documents
when their seat numbers are called.
A. Necessary B. Necessarily C. Need D. Necessity
10. The quality of the products manufactured by Robson‟s Electronics has remained
consistent since the firm‟s inauguration 25 years ago.
A. Remarkable B. Remarkably C. Remark D. Remarked
11. Although the president has made some irrational decisions lately, the board feels that he has
been reasonably in the past.
A. Cautious B. Cautiously C. Cautioning D. Caution
12. As in our video conference yesterday, Mr. Bailey will arrive in New
York on Friday, June 16 at 3pm
A. Discuss B. Discussion C. Discussing D. Discussed.
13. The Jones family has sent thanks to everyone in the office for the flowers that were
to their home.
A. Delivers B. Delivering C. Delivery D. Delivered
14. Turn of the medical researchers will work on the study to improve upon
the latest antibiotics to go on the market.
A. Collaboration B. Collaborates C. Collaboratively D. Collaborated
15. After considering her education and work background, the manager Ms.
Hannah for the position.
A. Recommend B. Recommending C. To recommend D. Recommended

16. The Sunstar Hotel which has expanded both in size and quality to attract
more tourists in coming years
A. Expects B. Expectation C. Expecting D. Expect
17. Though a great deal of money had , the project was a failure
A. Invested B. Be invested C. Been invested D. Invest
18. Public buildings throughout the country will soon a no-smoking policy
A. Implement B. Implementation C. Be implemented D. Implementing
19. It is a great pleasure to inform you that your company has as our new
supplier.
A. Selecting B. Selected C. Be selected D. Been selected
20. Because of a mechanical problem, the replacement parts could not _ by
tomorrow
A. Shipped B. Be shipped C. Be shipping D. Ship
21. A special luncheon will be held in honor of the sales department, which has
monthly sales goals
A. Reached B. Been reached C. Reaching D. Reached
22. According to our new policy, overnight camping in all national parks is no longer

A. Permits B. Permitting C. Permitted D. Permission


PHẦN 2/ DANH TỪ
A, PHÂN LOẠI CẤU TRÚC BỔ NGHĨA
- Các cấu trúc bổ nghĩa được chia làm 2 loại: cấu trúc bổ nghĩa đứng trước danh từ và cấu
trúc bổ nghĩa đứng sau danh từ
1, Loại 1: Cấu trúc bổ nghĩa đứng trước danh từ
*** Số từ/mạo từ + Adv + Adj + N

*** Số từ/ mạo từ + Adj + ... + Adj + N

2, Loại 2: Cấu trúc bổ nghĩa đứng sau danh từ


a, N + Cụm giới từ
b, N + Mệnh đề quan hệ đầy đủ
c, N + Mệnh đề quan hệ rút gọn
d, N + Đồng chủ ngữ
Các bước để làm bài tập phân loại động từ
- Xác đinh động từ trong câu, tránh bị động từ giả đánh lạc hướng
- Tìm chủ ngữ
- Xác định cấu trúc bổ nghĩa rồi phân tích
- Xác định sự hài hòa giữa chủ ngữ và động từ
B, MỘT SỐ BÀI TẬP THAM KHẢO
1. Nếu TO V or Ving là Chủ ngữ thì động từ phải ở số ít
EX: To spend more money on research and development by the consultant.
A. Has been suggested B. Have been suggested
C. Has suggested D. Have suggested
2. One of the + Danh từ số nhiều + Động từ số ít
EX: One of the most important things to achieve your goal to do your best.
A. Is B. Are C. Be D. Have been
3. A number + of + Danh từ số nhiều + V số nhiều
EX: A number of new employees expected to attend the orientation session
A. Is B. Are C. Be D. Has been
4. Câu mệnh lệnh không có chủ ngữ
EX: one of our financial assistants if you need any assistance.
A. Contact B. To contact C. Contacting D. Will contact
C/ THỰC HÀNH
1. Production of recreational vehicles significantly low this month
A. Are B. Have C. Is D. Be
2. New vacation policies which are under discussion expected to be
introduced early next month
A. Are B. Is C. Will D. To be
3. The for launching the new advertising campaign is posted on the bullention
board.
A. Schedule B. Scheduled C. Schedules D. Schedulers
4. Any late entries which _ submitted for the contest will not be considered
A. Is B. Will C. Being D. Are
5. Following safety precautions workers avoid any injury in the workplace
A. Helping B. Helps C. To help D. Help
6. If you require additional information, one of our sales representatives
A. Contacting B. Contact C. Contacts D. Contacted
7. The deadline for application for a in the marketing department is
st
July 1 .
A. Position B. Positions C. Positioning D. Positioned
8. The department‟s decision on whether or not _ Davidson Advertising is
dependent on the firm‟s ideas for product promotion.
A. Hire B. Hiring C. To hire D. Will hire
9. The _ made over the loudspeaker stated that the store would be closing
and customers were required to take their purchases to the nearest check-out counter.
A. Announcer B. Announced C. Announcing D. Announcement
10. The market consultants have used their analysis of the to recommend
development opportunities for the company.
A. Responses B. Responded C. Respond D. Responsively
11. Please be aware that annual of all employee receipts and expenses will be
made by the accounting department.
A. Reviews B. Reviewed C. Reviewer D. Reviews
12. The experience she gained working with the International Development team, along with
steady judgement, were the reasons for the recent promotion.
A. Her B. Hers C. She D. Herself
13. Mr. Jones would like to meet with the Human Resources Manager after
candidates.
A. Interview B. Interviewed C. Interviewing D. Interviews.
14. Payton Realty is known for its highly properties in and around the
metropolitan area.
A. Desire B. Desiring C. Desirable D. Desirous
15. This report contains the listing of the companies and organizations that
the company has been in negotiations with over the past year.
A. Distinguishably B. Distinguishable C. Distinguished D. Distinguish
16. The customer service division has been dealing with numerous regarding the
company‟s latest product, the M-700.
A. Complain B. Complaining C. Complainer D. Complaints
17. Board members have voted against the proposed of the European
offices.
A. Expansion B. Expands C. Expanded D. Expansive
18. Monica Scott has been nominated for the position of CPO because she has proven herself to
be a _ individual with strong accounting skills
A. Depend B. Dependable C. Dependability D. Depending
19. For the supervisor‟s position, ability to lead is an important _ of all successful
applicants.
A. Characteristic B. Characterize C. Characterizing D. Characteristically
20. Mr. Mc Crue‟s registration form for to the diploma program arrived the
day before the deadline.
A. Admission B. To admit C. Admit D. Admitted

Câu 21-23
Manage your business!
The 21. of making your business successful are here!
A. Secret B. Secrets C. Secretly D. Secrecy
Successfully managing a business 22. specific management skills in
addition to knowledge of key business practices.
A. Requiring B. Require C. Requirement D. Requires
Within this section you‟ll learn about leadership traits, decision-making skills, and how to
manage your employees.
Additionally, we‟ll help you learn a host of important topics which 23.
Marketing basics, setting prices, filing your business taxes, legal considerations,
forecasting for future growth, and financing options.
A. Including B. Include C. Includes D. Included

Câu 24-26
Enrollment policies
Requests to withdraw from a course must 24. in writing via email
before your allotted time has expired.
A. Receive B. Be receiving C. Receiving D. Be received
25. an email to registration@utuniversity.edu requesting a withdrawal.
A. Sending B. Send C. To send D. Sends
You will need to include your name, the course title and the course section number.
Following the proper withdrawal procedure will ensure an appropriate grade assignment.
No extension will 26. for this course.
A. Grant B. Have granted C. Be granted D. Have been granting
LESSON 02
PHẦN 1/ LÝ THUYẾT
A. KIẾN THỨC CƠ BẢN
Nhắc lại 1 số dấu hiệu nhận biết của các thì quan trọng trong tiếng anh
Tense Dấu hiệu nhận biết đặc trưng
Hiện tại đơn - Trạng từ chỉ tần suất
- Every
- Once a week, twice a week
Quá khứ đơn - Last/ ago/ in the past/ in + mốc thời gian trong QK
Hiện tại hoàn - Since  Hiện tại
thành - For  Hiện tại
- For the last ...
Tương lai - Next
- In + Khoảng thời gian
Câu điều kiện - Loại 1
- Loại 2
- Loại 3
Cách sử dụng - Cụm giới từ = Giới từ + (Adv) + Ving
V trong cụm gtừ
B. CHÚ Ý
1. Trong mệnh đề “that” theo sau những động từ chỉ yêu cầu hoặc những tính từ chỉ sự cần
thiết thì ta luôn dùng động từ nguyên mẫu.
Request – Recommend – Suggest – Insist that + S + Bare V/ Should V (+)
It is imperative/ essential/ important/ that + S + Not Bare V/ Should not V (-)
necessary/ vital/
.VD: It is imperative that the facilities in the plant be checked regularly
2. Các từ và cấu trúc đi với TO-V
a, Các cấu trúc theo sau bởi TO-V hay gặp trong bài thi Toeic.
Be willing to do Be supposed to do
Be able/ unable to do Be eligible to do
Be sure to do Feel free to do
So as to do = In order to do Hesitate to do
b, Các động từ theo sau bởi TO-V
Propose – Intend – Plan – Decide – Fail
3, Các danh từ theo sau bởi TO-V
Ability – Right – Way – Need
B3/ Các từ và cấu trúc theo sau bởi V-ING
1, Các cấu trúc theo sau bởi V-ING
Look forward to N/Ving Be committed to N/Ving Upon Ving
Have difficulty Ving Succeed in + Ving Be devoted to Ving/N
2, Các động từ theo sau bởi Ving
Consider – Avoid – Include – Keep + Ving
*** THỰC HÀNH KIẾN THỨC MỚI
Part V: Chọn từ thích hợp điền vào chỗ trống.
1. Early last week, the city council formally.............the downtown renewal project.
A. Approved B. Will approve C. Approves D. Has approved
2. After considerable effort, the company has succeeded ……………in the new environment
friendly home appliances.
A. Development B. Develops C. Developed D. Developing.
3. If commercial banks……………..mortage rates, many households will have difficulty in
repaying interest.
A. Will raise B. Raise C. To raise D. Raising
4. It is important for security guards ………………the security guidelines while they are on
duty.
A. Following B. To follow C. Followed D. Follow
5. Some technical support workers …………at the headquarters to learn how to operate the
new network access system over the past two months.
A. Trained B. Have trained C. Have been trained D. Are trained
6. The newly installed security system requires that every user……….the four-digit access
number.
A. Have B. To have C. Having D. Has
7. Construction workers are responsible for................their own equipment and tools.
A. Provides B. Providing C. Provided D. Provide
8. Failure to payment within three days of the final notice will result in immediate
legal action.
A. remit B. remitting C. remitted D. remittance
9. The Rothchild Group expects all of its staff members themselves in a
professional manner when dealing with clients.
A. conductor B. conducts C. to conduct D. be conducting
10. We recently received your new contact information and our client database
A. updating B. update C. updated D. are updated
11. Last month, marketing managers from all over the world at the London
Business Center for annual international marketing conference.
A. Convened B. convenes C. convening D. convene
12. After Saral Halton Manufacturing Co., She reduces unnecessary travel
expenditures from the budget in order to increase the company‟ net profits.
A. Join B. joins C. joined D. to join
13. Ms. Farriano recommended that we-------------Mr. Sidney as a representative of the company
the sales conference.
A. were selecting B. select C. were selected D. would selected
14. Some tourists were ---------- photos of the artwork on show in spite of the guide‟s request t
they not do so.
A. take B. taking C. taken D. took
15. Kirsten Gerhart, who------------the company‟s branch office in Washington, had a lunch meet
with the CEO yesterday.
A. run B. runs C. running D. were running
16. Ms. Luxton and her team of specialists----------a new marketing strategy since the beginning
the year.
A. are developing B. were being developed
C. have been developing D. would develop
17. All employees are expected to make the effort ----------- the company‟s new emerge
protocols.
A. learn B. learning C. learned D. to learn
18. The advertising task force would never have finished making the promotional materials if
Jefferson----------to help out.
A. has not volunteered B. was not volunteering
C. would not volunteer D. had not volunteered.
19. The supervisor in charge of the production team ------------- for next weekend‟s managem
workshop.
A. registered B. registration C. registering D. registry
20. Mr.Astle, who runs groceries in several areas,---------------a member of the London Society C
for over 30 years.
A. is B.was being C. has been D. is being

Câu 21-23
Want to sell your property?
Why only sell to local
buyers?
Intermational-Properties.com offers the first international private property sales service.
Our bilingual sales team enables the Intermational-Property.com network to sell your property
directly to our database of international clients.
If you 21………to sell property internationally, we will be the best choice.
A. Will plan B. Plans C. Plan D. Planned
We 22.................working as a specialist for international property over the past 20 years.
A. Were B. Have been C. Are D. Will be
By 23……………your property description to International-Property.com, your ad will
be promoted on your network until the sale of your property.
A. Provided B. Provision C. Providing D. Provided
GRAMMAR REVIEW LESSON 1 – 2
I. Chọn từ thích hợp điền vào chỗ trống.
1. The new managing director.................a more effective management structure.
A. Propose B. Proposed C. Proposal D. Proposing
2. The weather….........for tomorrow is partly cloudy with a chance of slight rain.
A. Forecasts B. Forecasted C. Forecasting D. Forecast
3. J&B Co. will be..............its new line of home appliances next month.
A. Introduce B. Introducing C. Introduces D. Introduced
4. Mr. Radford, the plant manager, recently ………….a tour of the production plant for
visitors from the Japan.
A. Conduct B. Conducted C. To conduct D. Will conduct
5.the personnel department directly if you have any questions regarding your
benefits.
A. Contacting B. Contact C. To contact D. Contacts
6. The monthly business meeting which is hosted by president………….an important role
in exchanging ideas.
A. Play B. Plays C. Playing D. To play
7. All requests for office supplies should be...................to Mr. Park by noon.
A. Submitting B. Submit C. Submitted D. Submission
8. The company announced that it plans to............several properties to expand its business.
A. Acquire B. Acquired C. Acquiring D. Acquires
9. Though many restaurants in the area offer an extensive dinner menu, the quality of
service.........................considerably.
A. Varies B. Vary C. Various D. Variety
10. To attract customers, discounted items should…...........at the front of the store.
A. Have displayed B. Displays C. isplaying D. Be displayed
11. The consulting firm is in the process of.................new marketing strategies for our new
products.
A. Establish B. Establishes C. Establishment D. Establishing
12. Mr. Evan has…............that the contract with Value Software Co. be renewed.
A. Suggest B. Suggestion C. Suggested D. Suggesting
13. After the current fiscal year, the management……………the construction of overseas
manufacturing plants.
A. Discontinued B. Will discontinue C. Have discontinued D. To discontinue
14. Free admission................for the city music festival are available upon request.
A. Ticket B. Tickets C. Ticketing D. Ticketed
15. The orphanage would like to thank people for their donations that were..................to it.
A. Delivers B. Delivering C. Delivery D. Delivered
16. Sales reports show that our brand of seasonal products……….very well right now
throughout the country.
A. Selling B. Sold C. Is selling D. Are sold
17. Before you…………..a new study, be sure to properly file the documents from the
previous one.
A. Begin B. Beginning C. Will begin D. Began
18. Managers should know the importance of….............a productive working environment.
A. Creation B. Creative C. Creating D. Creates
19. An increasing school age population…........that there will be a need for more teachers.
A. Imply B. Was implied C. Implies D. Have implied
20. Many companies have requested that regulations on foreign investments….........lifted
A. Are B. Is C. Be D. To be
Câu 21-23
Share your views on immigration
Are you feeling the impact of immigration?
What is life like for an illegal immigration who….............in the USA, and for others in the
communities where he or she works or lives?
21. A. Live B. Lives C. Living D. Life
We‟re interested in hearing your experiences. Send comments to editor@immigration.com
or fax to 703-854-2053.
Please......................your contract number, city and state for verification purposes.
22. A. Included B. Inclusion C. Include D.
Includes Comments will......for an ongoing conversation in letters and online.
23. A. Consider B. Have considered C. Considering D. Be considered
Câu 24-26
Community Events Sponsorship guidelines
Thank you for your interested in community events sponsorship, please see the addresses
below for submission of sponsorship requests, Proposals will be evaluated until March 1
for that particular year.
All…................for this year have been collected.
24. A. Funds B. Funding C. Fund D.
funded Proposals for the next year….. .evaluated beginning October 7th.
25. A. Have B. will be C. Is D. will
……………a proposal for event sponsorship, please send it to:
26. A. Submit B. Submission C. to submit D. Submitted
Community Events
Sponsorships 435 N. Michigan
Ave
LESSON 03
A/ COMMON VERBS IN TOEIC
1. Renew Renewal (n)
Renew/ sign/ terminate a contract
2. Submit Submission (n)
Submit the data to Ray
3. Reserve Reservation (n)
Reserve the right to refuse
4. Retain Retention (n)
Retain competent employees
5. Represent Representative (n)
Representation (n)
Represent our company
6. Confirm Confirmation (n)
Confirm/ make / cancel a reservation
7. Attribute
Attribute economic improvement to the government‟s policy
8. Attract Attractive (n)
Attraction (n)
Attract many investors
9. Introduce Introduction (n)
Introduce the competitors‟ products
10. Review
Review the annual report
*** Vocabulary Review
1. We plan to [prevent, renew] our service contract with INC Shipping.
2. The company [reserves, hesitates] the right to revise the contents of the journal.
3. JP Associates will [intend, represent] our company for all future negotiations.
4. The new exhibition is expected to [present, attract] many visitors.
5. The products division will [introduce, enclose] a new microwave oven.
6. Please make sure that the hotel reservation has been [confirmed, delivered].
7. Employees are reminded to [handle, retain] all receipts during business travel.
8. The company [attributed, contributed] its success to dedicated employees.
9. You need to carefully [install, review] the terms of the contract.
10. New employees should [submit, implement] personnel forms to their supervisor.
B/ HOW TO USE COMMON VERBS IN TOEIC
1. Attend
Participate in
2. Offer (Offer A B)
Provide (Provide A with
B)
3. Suggest (Suggest –ing)
Propose (Propose to do sth)
4. Inform (Inform A of B)
Tell (Tell A about B)
5. Spend (Spend A on ~ing)
Invest (Invest A in sth)
6. Remind
Remind A of B
Remind A to do
A is reminded to do
7. Ask
Ask A to do
A is asked to do
8. Have
Have + sb do sth
Have + sth done by sb
9. Prohibit
Prohibit A from ~ing
A is prohibited form ~ing
*** Vocabulary Review
1. The company has [spent, invested] a lot on developing environment-friendly vehicles.
2. The consultant has [proposed, suggested] adoping more aggressive marketing strategies.
3. All new employees have to [attend, participate] the orientation on company policies.
4. My secretary has [informed, told] all board members of the director‟s sudden resignation.
5. Kathy [was reminded, was remained] that a full refund will be given only if the product is
returned within 10 days of purchase.
6. The organizer will [offer, provide] all participants with a detailed conference schedule.
7. Customers are asked [to join, joining] the musical festival at no charge.
8. To protect the national park, the government has prohibited hikers [from, of] staying
overnight.
9. Many customers want to have their credit card [renew, renewed] through the phone.
C/ COMMON NOUNS IN TOEIC
1. Productivity Produce (v)
Production (n)
Improve productivity
2. Performance Perform (v)
Performance review
3. Contribution
Make a contribution to + sth = Contribute to + sth
Contribute A to + sth
4. Requirement Require (v)
Tax requirement
5. Charge
Handle charges
6. Attention
Draw/ catch one‟s attention
7. Satisfaction Satisfy Satisfied (a)
To one‟s satisfaction
Meet the customers‟ satisfaction/ expectation/ needs
8. Complaint Complain (v)
Complaint form Make a complaint
9. Need
A need to do
10. Apology Apologize for
Accept an apology for the delay
*** Vocabulary Review
1. Please be aware that shipping [charges, fares] are not included in the total price.
2. Our technicians try to do their best to offer customers complete [appointment, satisfaction]
3. Please accept our [apologies, increase] for the inconvenience caused by the construction.
4. Increasing staff [analysis, productivity] is the new president‟s main concern.
5. Only those applicants who meet the [requirements, growth] for the position will be
interviewed.
6. She received a standing ovation for her outstanding [performance, approval].
7. You can contact my secretary if you have any urgent business that needs my [results,
attention].
8. There is a growing [need, compensation] to develop alternative energy sources.
9. We want to express our thanks for Dr. Kim‟s [contributions, shortage] to our research.
10. Our service representatives are trained to handle [complaints, forecast] from customers in a
professional manner.
D/ HOW TO USE COMMON NOUNS
1. At one‟s expense
Expense (n) Expensive (a)
2. At one‟s earliest convenience
Convenience (n)
Convenient (a) Conveniently (adv)
3. In compliance with (=in observance of)
Compliance (n) Comply with
4. In honor of (= In celebration of)
Honor (n)
(v)
5. Reserve the right to
do
6. Be on schedule
Schedule (n)
(v)
7. In conjunction with (= In combination with)
Conjunction (n)
8. Take advantage of
Advantage (n)
9. Go into effect (= Out into practice)
Effect (n) Have an effect on
10. Take safety
precautions Precaution (n)

*** Vocabulary Review


1. Restaurant employees should always take [precautions, proportion] to prevent fires.
2. This site gives customers detailed reviews of products [in agreement with, in conjuction with]
many informative resources.
3. The current price will remain [in effect, in writing] until further notice.
4. Smoking is no longer allowed in the building [in compliance with, in combination with] the
management‟s policy.
5. Any illegally parked cars will be towed away at the owner‟s [policy, expense].
6. Please return the survey to our office at your earliest [convenience, invitation].
7. Teachers can take [profit, advantage] of an additional 10% discount for educators.
8. A special award will be given to Mr. Lennon [in honor of, in excess of] his dedication.
9. The development plan has been proceeding [on schedule, on request].
10. Please be aware that the company reserves the [right, facility] to change the prices.

E/ PRACTICE Test 1 | Động từ và danh từ


I. Chọn từ thích hợp điền vào chỗ trống.
1. The bank has confirmed that it will take necessary measures to another computer
system failure.
A. Upgrade B. Encourage C. Prevent D. Improve
2. The management decided to lay off some temporary workers in an effort to reduce
.
A. Values B. Expenses C. Fines D. Refunds
3. To apply for the advertised position, please an application with the necessary
documents to the human resources department.
A. Comply B. Submit C. Urge D. Accommodation
4. The marketing seminar scheduled for tomorrow is for employees who have been
with the company less than a year.
A. Intended B. Based C. Agreed D. Invited
5. If you want to reschedule your , please give us at least 24 hours advance notice.
A. Appointment B. Position C. Assignment D. Subscription
6. All visitors are required to photo identification before entering our research center.
A. Notify B. Assign C. Permit D. Present
7. In a preview, seats in the front row are usually for VIP guest.
A. Chaired B. Reserved C. Performed D. Submitted
8. Ultra Discount Stores Inc. reserves the to change the free gift items wihout prior
notice.
A. Importance B. Selection C. Right D. Goal
9. While the service center is closed for the renovation, all questions concerning repairs will be
by the customer service department.
A. Expired B. Handled C. Replied D. Attending
10. Our policy clearly states that the board of directors should an outside auditor to
oversee our company „s accounting practices.
A. Deposit B. Appoint C. Predict D. Operate
11. Business Solution, one of the leading international business magazines, has in
over 30 countries.
A. Subscribers B. Spectators C. Witnesses D. Participants
12. Because of the large number of people, we had to wait for more than 3 hours to
tickets for the baseball game.
A. Support B. Purchase C. Achieve D. Replace
13. We are delighted that your generous have made it possible for us to support
children from low income families.
A. Acquisitions B. Contributions C. Additions D. Difference
14. Business owners want to hire employees with ambition, energy and a desire to make a
.
A. Complaint B. Decision C. Living D. Difference
15. Our service representatives commited to providing quality service for customers.
A. Retain B. Remember C. Reimburse D. Remain
16. Our current subcription rates will be effective until further .
A. Notice B. Noticeable C. Noticed D. Notices
17. The board of directors unanimously agreed to your consultant contract for an
additional two-year period.
A. Redeem B. Renew C. Rehire D. Recall
18. Customers may request a refund or only if the merchandes is still unused.
A. Replacement B. Complaint C. Receipt D. Promotion
19. One of the greatest _ of online bookstores over traditional ones is saving time and
money.
A. Incomes B. Advantages C. Interests D. Designs
20. If you need any information about the opening, please do not _ to contract me.
A. Provide B. Qualify C. Hesitate D. Compete
Câu 21 ~ 23
To : All staff
From: Dennis Roy
Subject: Safety precaution
This is to remind everyone that the quarterly safety 21. on our safety facilities
is just around the corner.
21. A. Advance B. Alternative C. Consent D. Inspection
I would like each of you to check all the facilities and other equipment. Special attention
should be given to fire extinguishers. Please make sure those extinguishers in your office are in
good working 22. .
22. A. Conditional B. Condition C. Conditions D. Conditioned
If not, please 23. me so I can arrange for maintenance as soon as possible.
23. A. Explain B. Speak C. Notify D. Mention
If you have any questions or concerns, please come see me.
Thank you.
Dennis Roy
Facility Manager
Câu 24 ~ 26
Looking for free shipping?
Free shipping applies only to items marked with the words “free shipping” at the time of
24. .
24. A. Purchase B. Purchasing C. Purchased D. Purchases
Free shipping applies to select items in the home décor, lighting, rugs, sipcovers,
furniture, appliances, pets, sports, electronics, baby and kids categories at Target.com as
indicated in the product description page. Offer does not include 1-day or 2-day shipping and
handling 25. and it may not be combined with other promotional offers or discounts
or applied to previous orders.
25. A. Charges B. Figures C. Values D. Fares
Offer 26. December 11, 2007.
26. A. Violates B. Invalidates C. Expires D. Exceeds
LESSON 04
A/ ĐẠI TỪ
PHẦN 1: LÝ THUYẾT
1, Đại từ sở hữu
MINE – YOURS – OURS – THEIRS – HIS – HERS – ITS
- Đại từ sở hữu = Tính từ sở hữu + Danh từ được nhắc tới trước đó, tránh tình trạng
lặp lại từ trong câu.
Vd: Because Mr Laurent completed his department‟s financial budgets ahead of schedule,
he offered to help Ms Kong finish
A. hers B. her C. she D. herself
2, Đại từ chỉ định: THAT/ THOSE
- That thay thế cho danh từ số ít, those thay thế cho danh từ số nhiều
- THAT/THOSE thường được theo sau bởi giới từ OF
Vd1: His point of view on investment is similar to of an expert.
A. That B. Those C. It D. Them
Vd2: A company uses a brand to distinguish its product from of another
company.
A. That B. Those C. It D. Them
- Ngoài ra, đại từ THOSE còn có thể đi cùng với đại từ quan hệ WHO, Trong trường
hợp này “Those who” có nghĩa là những người.
Vd3: An extra customer service training session will be held before work tomorrow morning
for-------------who were absent from Friday‟s seminar.
A. them B. those C. these D. which
3, Đại từ phản thân là đại từ đƣợc sử dụng để phản ánh chính bản thân ai đó
MYSELF – YOURSELF/YOURSELVES – OURSELVES –
THEMSELVES – HIMSELF – HERSELF – ITSELF
- Đại từ phản thân được sử dụng để phản ánh chủ ngữ và tân ngữ là một, thường
đứng sau động từ hoặc giới từ.
VD1: When I am alone, I often talk to myself
- Đại từ phản thân đứng sau giới từ
 BY + ĐTPT: Một mình
VD: I completed my homework by myself
 FOR + ĐTPT: Cho chính ai
- Đại từ phản thân đứng ngay sau chủ ngữ hoặc đứng ở cuối câu để nhấn mạnh chính
ai làm gì đó
VD: I made coffee myself = I myself made coffee
Vd1: The marketing department decided to perform focus group analysis
because hiring a new consulting firm was outside their budget
constraints.
A. Myself B. Herself C. Yourselves D. Themselves
TOEIC FOCUS ---- TOEIC BUILDING-----Smart Home for Toeic

Vd2: A responsible financial planner gives clients enough advice to help them make a
right decision for
A. They B. them C. their D. themselves
Vd3: Mr. Grant also reviewed the budget proposal ----------- before forwarding it to the
finance director for final approval.
A. He B. his C. him D. himself

4, Phân biệt ANOTHER/ THE OTHER


- Another: 1 đối tượng trong nhóm có từ 3 đối tượng trở lên, thường sử dụng trong
cấu trúc “from one …. to another”
Vd: There is a fee to transfer money from one account to another
- The other: Cái còn lại
Vd: Of the three meeting rooms, two are already reserved while the other is available.
PHẦN II. THỰC HÀNH
1. Most of the start- up companies spend a relatively high percentage of--------budgets on
advertising costs.
A. They B. Their C. Them D. Theirs
2. Because all members were busy doing their work, the sales manager had to finish the
sales report by ---------
A. Him B. His C. Himself D. He
3. Priority will be given to ------------- who do not have any previous experience in this
field.
A. They B. Those C. Him D. That
4. Please be sure to keep all receipts and submit ----------with your claim for
reimbursement.
A. They B. Their C. Them D. Themselves
5. You need to type the password when you want to send a document from your
computer to--------------.
A. Other B. Each other C. The other D. Another
6. The marketing department decided to perform focus group analysis
because hiring a new consulting firm was outside their budget constraints.
A. Myself B. Herself C. Yourselves D. Themselves
7. Mrs. John left a message to the postal service department so that they could notify
when the package arrived.
A. Her B. Herself C. She D. Hers
8. The employees were able to network more efficiently as began
using the new computer system.
A. Themselves B. Them C. Their D. They
9. To receive credit for relocation expenses, employees have to make copies of all
receipts and submit with the signed request.
A. They B. Their C. Them D. Themselves
10. An extra customer service training session will be held before work tomorrow morning
for who were absent from Friday‟s seminar.
A. them B. those C. these D. which

146
B/ TÍNH TỪ
PHẦN 1. LÝ THUYẾT
1, - (HĐT) + Adj + N  (HĐT) + Adv + Adj + N
EX: A big house, my beautiful girlfriend, …
Vd1: The experience she gained working with the International Development team, along
with steady judgement, were the reasons for the recent promotion.
A. Her B. Hers C. She D. Herself
Vd2: Payton Realty is known for its highly properties in and around the
metropolitan area.
A. Desire B. Desiring C. Desirable D. Desirous
Vd3: Board members have voted against the proposed of the
European offices.
A. Expansion B. Expands C. Expanded D. Expansive
Vd4: The customer service division has been dealing with numerous
regarding the company‟s latest product, the M-700.
A. Complain B. Complaining C. Complainer D. Complaints

2, Tính từ đứng sau Linking verb trong cấu trúc: SUBJECT + LINKING VERB +
ẠDJECTIVE
- Một số Linking verb thường gặp: Be/ Become/ Remain/ Seem/ Feel/ Look.
Vd1: The consulting firm is that the new advertising campaign
will significantly increase market share.
A. Optimist B. Optimistic C. Optimism D. Optimistically
Vd2: The report is thorough and will enable the product development
team to understand the recent upturn in the electronics industry.
A. Extreme B. Extremity C. Extremely D. More extreme
Vd3: The technical team has assured that the new computer system will be
functional by the end of the month, when annual reports are due.
A. Complete B. Completing C. Completely D. Completion

3, Tính từ có thể xuất hiện trong cấu trúc: SUBJECT + TRANSITIVE VERB +
OBJECT + ADJECTIVE
- Có một số động từ thường gặp đi theo cấu trúc trên là: CONSIDER/ FIND/ MAKE/
DEEM/ DRIVE
Vd1: Although many people deemed it , the Planto Corporation was able to
double its sales in its second year of business.
A. impossible B. impossibility C. impossibly D. possible
VD2: Some people consider science books interesting.
A. Extreme B. extremely C. Extremeness D. Extremist
PHẦN 2. THỰC HÀNH
1. Eva Consulting earned the (complete/ completion/ completely) trust of its clients
through its thorough analysis.
Dịch + Giải thích:
2. The first draft needed (extensive/ extension/ extensively) revision to meet expectations.
Dịch + Giải thích:

3. Investors remain (optimism/ optimistic/ optimistically) about the stock


price. Dịch + Giải thích:

4. Good trainers will make your workout more (effect/ effective/


effectively). Dịch + Giải thích:

5. You need to get as (many/ much) information as possible.


Dịch + Giải thích:

6. Next generation cars will emit fewer (pollution/ pollutants).


Dịch + Giải thích:

7. If you reserve an airplane ticket two months earlier, you will get a (substantiate/
substantial/ substance/ substantially) discount.
Dịch + Giải thích:

8. All companies should balance their (economic/ economy/ economist/ economically)


progress with social responsibility.
Dịch + Giải thích:

9. The country is becoming increasingly (relied/ reliant/ reliance/ relying) on petroleum


imported from Russia.
Dịch + Giải thích:

10. All employees should be (familiarity/ familiarly/ familiar/ familiarize) with the new
safety regulations.
Dịch + Giải thích:

11. Mr. Viera will face many new (challenge/ challenges/ challenging/ challenged) in his
new position.
Dịch + Giải thích:
12. Despite a few negative (comments/ comment/ commenting/ commented) from
competitors, our new sedan model is selling well.
Dịch + Giải thích:

C/ TRẠNG TỪ
PHẦN 1. LÝ THUYẾT
*** Vị trí và chức năng của trạng từ
1/ Đứng sau động từ để mô tả cách thức hành động đƣợc tiến hành nhƣ thế nào
Cụ thể như sau:
 Nội động từ + Trạng từ
 Ngoại động từ + Danh từ + Trạng từ: He reads a book quickly.
Vd1: Human Resources managers should have excellent speaking and conflict resolution
skills to communicate with a variety of employees.
A. Effect B. effective C. effectively D. effectiveness
Vd2: A specialist mediator was brought in to help the two parties solve their dispute over the
real estate constract---------
A. quick B. quickly C. quicker D. quickest
2/ Đứng trƣớc động từ ở thì hiện tại đơn hoặc quá khứ đơn
Vd3: The finance director of the government health care agency ------------- accepted bribes
from the pharmaceutical companies.
A. allegedly B. alleged C. allege D. allegations
3/ Đứng giữa trợ động từ/ động từ khuyết thiếu và động từ chính
Vd4: Melbourne Super‟s fresh fruit and produce are ----------- certified as organic and free of
all additives and preservatives.
A. official B.office C. officially D. officiate
Vd5: The manufacturing firm, which had been searching for opportunities to expand its
operations, has------------acquired a new factory in Newark.
A. final B. finalize C. finalist D. finally
4/ Đứng đầu câu để bổ sung ý nghĩa cho cả câu
Vd6----------, the IT technicians should be able to recover most of the data that was lost when
the serve malfunctioned.
A. Fortune B. Fortunate C. Fortuitous D. Fortunately
5/ Trạng từ đứng trƣớc tính từ để nhấn mạnh cho tính từ đó
Vd7: The changes to the company‟s compensation plan have been ------------- effective in
improving the employees morale.
A. remarkably B. remarked C. remarks D. remarkable
6/ Trạng từ đứng trƣớc trạng từ
Vd8: My sister-in-law can sing----------------well.
A. Extreme B. extremely C. Extremeness D. Extremist
*** Một số cụm nội động từ - trạng từ phổ biến
- Travel regularly - Grow rapidly
- Increase dramatically - Decrease slightly
- Work collaboratively - Meet frequently with
- React calmly to delays - Rely heavily on
- Speak clearly to the audience - Communicate easily with
- Leave promptly for
PHẦN 2: THỰC HÀNH
1. The workshop was (origin/ original/ originally) scheduled to begin at
9am. Dịch + Giải thích:

2. Many of our new employees have not yet been trained (adequate/ adequacy/
adequately).
Dịch + Giải thích:

3. Solar power is an energy source that is (economy/ economically/ economic)


beneficial. Dịch + Giải thích:

4. The PR manager is an (absolution/ absolute/ absolutely) ideal position for someone


who enjoys dealing with the press.
Dịch + Giải thích:

5. The entire division worked very (hard/ hardly) to complete the assignment on
time. Dịch + Giải thích:

6. The proposal could not be accepted because it was submitted


(late/lately). Dịch + Giải thích:

7. At Vento Electronics, service representatives handle customer suggestions (attention/


attentive/ attentively/ attentiveness).
Dịch + Giải thích:

8. Thompson Consulting will (shortly/ short/ shorter/ shorten) be opening a branch office
in Rio.
Dịch + Giải thích:

9. Mr. Dumas is (high/ highly/ higher/ highest) regarded because of his passion and
creativity.
Dịch + Giải thích:
10. The project manager was (extreme/ extremes/ extremely/ extremity) proud of his team
for completing the proposal successfully.
Dịch + Giải thích:

11. The machine looks new because it has (hard/harden/hardly/hardness) been


used. Dịch + Giải thích:

12. Rumors about the health of the president have been circulated (late/ lately/ lateness/
latest)
Dịch + Giải thích:

13. Sam, the marketing director, has been known for presenting many
(creation/creator/creating/creative) advertising ideas.
Dịch + Giải thích:

14. Evaluation forms are (convenience/conveniently/convenient/convene) placed outside


the doors so that you can pick them up as you leave.
Dịch + Giải thích:

15. This new automatic ordering system is to help customers place their orders more
(quick/quickly/quickness/quickest) regardless of time and location.
Dịch + Giải thích:

16. Patrick Simmons is (Responsible/ Responsibly/ Responsibility/ Responsibilities) for


both recruiting new sales personnel and training them.
Dịch + Giải thích:

17. Because the train leaves (prompt/promptness/promptly/prompted) at 7:00, ticket


holders should board before 6:50.
Dịch + Giải thích:

18. Should you need any technical assistance with the company system, please notify
technical support staff in a (time/timely/timing/times) manner.
Dịch + Giải thích:

19. Mr. Osman travels (regularly/regularity/regular/regulate) for business to find out new
investment opportunities throughout the world.
Dịch + Giải thích:

Câu 20-22
Welcome
Our mission is to promote and protect the health and 20-----------of all residents
A. Safe B. Safely C. Safety D. Safer
From birth throughout life, we can help enrich and protect the lives of the people of 21.----
----- state.
A. We B. Ours C. Us D. Our
We offer programs that deal with complex social, economic and individual issues.
22-----------range from Medicaid to food stamps, child protection to disease prevention.
A. They B. He C. It D. There
Last year, we helped almost 340,000 people in our state, one out of every four state
residents.

Câu 23-25
The keys to Great Sales Presentations
If your business depends 23...........................on selling product or services to other firms,
A. Heavy B. Heavily C. Heaviness D.
Heavier Then you and your staff need to make great sales presentations.
Here‟s how to make presentations that show 24. ……………… clients exactly what your
company can do for them.
A. Prospectively B. Prospect C. Prospects D. Prospective
 Rehearse every aspect of your presentation.
 Read in front of a mirror to practice eye contact.
 Rehearse in front of a friend or colleague who can offer 25. …………………
criticism.
A. Construction B. Constructive C. Constructively D. Construct
LESSON 05
A/ Hệ thống lại kiến thức tổng quan về so sánh của tính từ
PHẦN 1. THỰC HÀNH
- Tính từ ngắn là tính từ có một âm tiết, các tính từ có 2 âm tiết kết thúc bằng y/ly,
một số tính từ đặc biệt như: Narrow, clever, quiet, modern
- Tính từ dài là tính từ có 2 âm tiết trở lên

So sánh bằng So sánh hơn So sánh hơn nhất


Ý nghĩa A=B A>B Nhất
Cấu as as * adj + er + than
trúc - He is as tall as (Adj ngắn) * the + adj + est
his brother - He is taller than me - He is the tallest
- He is as * more + adj + than the class.
handsome as his (Adj dài) * the most + adj
brother - He is more intelligent than - He is the most handsome
her in the class

Các -Good  Better  The best


trƣờng -Bad  Worse  The worst
hợp đặc -Far  Farther  The farthest
biệt Further The furthest
- Many  More  The most
- Little  Less  The least
Chú ý - Trạng từ nhấn mạnh cấu trúc so sánh
 Trạng từ nhấn mạnh so sánh hơn: Much, far, even, still, a lot
- The + số thứ tự + adj so sánh hơn nhất
- S1 + Linking verb + as + Adj + as + S2
Verb Adv
- The +-----------+ Noun  Chỗ trống ưu tiên tính từ so sánh hơn nhất
PHẦN 2: LÝ THUYẾT
1. The demand for our new SUV vehicle is (much / very) bigger than we anticipated.

Dịch + giải thích:

2. Producing high quality products is more important (as/ than) making a profit.

Dịch + giải thích:

3. Of all the subway lines that serve downtown, Blue Line is the (easier/easiest) to

access. Dịch + giải thích:

4. Developing the satellite camera is the (more/ most) challenging project our laboratory
has ever undertaken.
Dịch + giải thích:

5. Sleeping on the sofa is not as (comfortable/ comfortably) as sleeping on a bed.

Dịch + giải thích:

6. For a fired monthly charge, customers can make as (many/ much) telephone calls as
they wish.

Dịch + giải thích:

7. Sky one Airlines canceled more flights (as/ while/ than/ whether) any other airline in
September.

Dịch + giải thích:


8. LCD Ns are much (so affordable/ more affordable/ most affordable/ as affordable) now
than 3 years ago.

Dịch + giải thích:

9. Our sales team will provide our customers with the ( widely/ widest/ wide/wider)
selection of products on the market.

Dịch + giải thích:

10. Breaking into the Japanese market is the (challenging/ more challenging/ most
challenging/ more challenged) task Global Elections has never attempted.

Dịch + giải thích:


11. This year‟s profit is not as (greater/ great/ so great/ greatest) as last

year‟s. Dịch + giải thích:

12. Fiber optic cables make it possible to use a communication network as efficiently (as
/at / than/ to) possible.

Dịch + giải thích:

B. PHÂN TỪ
PHẦN 1: LÝ THUYẾT
1, Câu hỏi
Điền dạng đúng của động từ cho sẵn vào cấu trúc
- The companies (Participate)
- The schedule (Attach)
2, Cách phân tích câu hỏi
- Từ cần điền vào chỗ trống ở đây là tính từ. Tuy nhiên, từ cần điền vào chỗ trống ở
đây là động từ nên đáp án sẽ là tính từ phân từ. Để xác định đáp án là phân từ hiện
tại hay phân từ quá khứ, ta xem danh từ giống như là chủ ngữ của động từ cho sẵn.
Nếu mang nghĩa chủ động, từ cần điền là phân từ hiện tại, nếu mang nghĩa bị động
từ cần điền là phân từ quá khứ.
- Đặt companies làm chủ ngữ, participate làm động từ
 Nghĩa chủ động: Công ty tham gia hợp lý hơn
 Đáp án là phân từ hiện tại: Participating
 The participating companies nghĩa là các công ty tham gia
- Đặt schedule làm chủ ngữ, attach là động từ
 Nghĩa bị động: Lịch trình được đính kèm hợp lí hơn
 Đáp án là phân từ bị động: Attached
 The attached schedule nghĩa là lịch trình được đính kèm
3, Hai loại phân từ chỉ cảm xúc
- Ving được dùng để chỉ bản chất của con người/sự việc
- Ved được dùng để chỉ trạng thái tạm thời, thường là con người chịu tác động của
bên ngoài
 The test result was very disappointing
 He was very disappointed at the test result
- Các phân từ chỉ cảm xúc thƣờng gặp
Dạng Ving Dạng Ved
- Disturbing - Disturbed
- Troubling - Troubled
- Interesting - Interested
- Exciting - Excited
- Disappointing - Disappointed
- Encouraging - Encouraged
- Satisfactory - Satisfied
- Fascinating - Fascinated

PHẦN 2: THỰC HÀNH


1. Wind Ltd is an internationally (recognizing / recognized) advertising

firm. Dịch và giải thích:

2. Customers can get a copy of the (updating/ updated) version of our product brochure.

Dịch và giải thích:

3. The government is aware of the (rising/ rised) demand for energy.

Dịch và giải thích:

4. The latest market analysis shows (culminating/ culminated) consumer interest in


healthy foods.

Dịch và giải thích:


5. The cooling fan on the computer runs only when (needing/ needed).

Dịch và giải thích:

6. When (mailing / mailed) breakable goods, you should pack them carefully.

Dịch và giải thích:

7. All employees should follow the office procedures (proposes/ proposing/ proposed/
propose) by the management.

Dịch và giải thích:

8. Aerospace, Inc. is developing a new (will be monitored/ monitoring/ to monitor/


monitored) system to detect the status of an engine.

Dịch và giải thích:

9. Because of its (rising/ rose/ arisen/rise) rental cost, the company has decided to move
to another building.

Dịch và giải thích:

10. When (apply/ applying/ applied/ to apply) for the position, candidates should submit
the application form with a résumè.

Dịch và giải thích:

GRAMMAR REVIEW LESSON 4-5


Part V: Chọn từ thích hợp điền vào chỗ trống.
1. Several samples of office uniforms will be on display in the lobby for the
of the week.
A. Remaining B. Remainders C. Remainder D. Remain
2. Safety experts stress that inspections of the hospital facilities will ensure
safely and proper functioning
A. Frequent B. Frequency C. Frequently D. Frequents
3. The president of Alliance Investment Co announced the cancellation of
its plan to expand its operation in Japan
A. regretfully B. Regretful C. regretting D. regretted
4. The committee informed Mr. Yamamoto that_ application cannot be
accepted because he didn‟t submit the required documents,
A. He B. Himself C. His D. Him
5. World Travel Co. is currently offering all new two free round-trip airline
tickets.
A. Client B. Customer C. Patron D. Clients
6. Whenever the company introduces new products, its employees try the products
before marketing them.
A. Themselves B. Himself C. Herself D. Myself
7. Taking a subway is faster any other available method to get to the
airport.
A. As B. Than C. More D. Most
8. Workers who work with hazardous materials should remember to be
and always wear protective clothing.
A. Cautious B. Caution C. Cautiously D. Cautiousness
9. Loan applications must be filled out before being submitted to our loan
office.
A. Complete B. Completes C. Completely D. Completed
10. Travelers are advised to make reservations for their trip before the rate
increase.
A. Predict B. Predicting C. Predicted D. Prediction
11. Of the two final candidates, one is reluctant to work abroad so will be
considered for the position
A. Others B. The other C. Another D. Other
12. Ambition and diligence are the most important of all successful
businessmen.
A. Characteristics B. Characteristically C. Characterizing D. Characterize
13. The system engineers work more efficiently as began using the new
technology.
A. Themselves B. Them C. Their D. They
14. To contend with increasing orders, we put an advertisement to hire
help.
A. Addition B. Additions C. Additional D. Additionally
15. We are working with city officials to identify potential sites for the
public library.
A. Proposing B. Proposed C. Propose D. Proposal
16. SIC Manufacturing Co. which has a 20% market share has become our_
competitor.
A. Strongly B. The strongest C. Strongest D. Strength
17. Researchers from several chemical research institutions which will work
on this study will have a meeting
A. Collaboration B. Collaborates C. Collaboratively D. Collaborated
18. Our hotline service is available for _who are not satisfied with our
products and services.
A. He B. That C. Those D. They
19. Financial experts agree that the nation‟s trade deficit is deeply and will
have a negative impact on the entire economy.
A. Troubled B. Trouble C. Troubling D. Troubles
Câu 20 – 22
Ready to Market High-speed Internet Service
The world‟s 20. retailer, Best Buy Stores Inc., is ready to market new high-
speed Internet service. The high-speed internet will be available at 570 Best Buy stores in
13 states.
20. A. Larger B. Largest C. Largest D. Largely
Best Buy Store Inc. has begun marketing the service at 21. Best Buy
Connection center locations
21. A. Their B. His C. Her D. Its
The Connection Centers are 22. areas within Best Buy stores that sell various
broadband and wireless services to customers
22. A. Specialize B. Specializing
C. Specialized D. Specialization
Starting next month, trained Best Buy Connection Center employees will be able to help
customers determine whether the service is available in their neighborhood as well as help
them place orders for the high-speed internet service.

Câu 23-25

Online Advertising

Online advertising is one of the most powerful components of any media campaign, and
Adjournal is the 23. fit for companies of all sizes.
23. A. Perfection B. Perfectly C. Perfect D. Perfected
No other site delivers a more influential local business audience 24. Adjournal,
24. A. Most B. Than C. As D. More
With more than 3.2 million registered users, Adjournal has proven 25. to be
an
25. A. Them B. Itself C. It D.
Themselves industry leader among local business news websites.
LESSON 06
VOCABULARY (2) – ADJECTIVES/ADVERBS
A/ BASIC ADJECTIVES IN TOEIC
1. Additional Additional funds Add (v)
Addition (n)

2. Limited A limited number of Limit (V)


attendees Limitation (n)
3. Defective Defective merchandise Defect (n)
4. Promising The most promising Promise (v)
candidate
5. Convenient (- inconvenient) Convenience (n)
Convenient access Conveniently (adv)
Be conveniently located
6. Available (- unavailable) Availability (n)
A duplicate of the contract
is available.
7. Valuable A valuable member Value (n)
8. Close Close examination Closely (adv)
Close the deal Come close to + noun
9. Likely Kathy is likely to finish the Like (v)
project.
10. authorized Authorize the purchase Authorize (v)
Authorized representative Authority (n)
*** Vocabulary review
1. Employees are the most (valuable, upcoming) assets in our company.
2. We guarantee a full refund for any (prior, defective) items.
3. There is a need to hire (additional, comprehensive) programmers to finish the project
on time.
4. Ms. Weiss is one of the most (similar, promising) members in the marketing team.
5. Technological advances are (likely, positive) to help improve staff productivity.
6. To better serve our customers, we run more than 100 (authorized, specific) service
centers.
7. Shuttle services to the airport will be (sincere, available) starting from next Monday.
8. Because of its (convenient, temporary) location, the hotel attracts a lot of business
travelers.
9. To run the assembly line more effectively, (close, accessible) supervision is
recommended.
10. Items on sale are (limited, necessary), so you should take quick action.

B/ BASIC ADVERBS IN TOEIC


1. Finally A new advertisement was Final (n)
finally released.
2. Currently (= presently) Current (a)
Be currently under construction
3. Directly Report directly to the director Direct (v)
(a)
4. Promptly Leave promptly at 7 Prompt (a)
5. Completely A completely independent Complete (v)
agency Completion (n)
6. Highly Financial incentives are highly High (a)
recommended.
7. efficiently Run relatively efficiently Efficient
Efficiency
8. relatively Be relatively expensive Relative (a)
9. collaboratively Work collaboratively Collaborate (v)
Collaborative (a)
10. significantly (=considerably, substantially) Significant
Be reduced significantly significance
*** Vocabulary review
1. To apply for a loan, please fill out the application form (completely, necessarily).
2. It is important for service workers to respond (only, promptly) to customer complaints.
3. You can contact me (directly, securely) in case of emergency.
4. All department heads are reminded to work (collaboratively, exclusively) on the
project.
5. We have a (appropriately, relatively) stringent policy for all students.
6. The two companies (finally, significantly) agreed to sign the merger contract.
7. The consultant has been asked to find ways to run the plant more (efficiently,
absolutely).
8. We received many applications for the position from (highly, correctly) qualified
candidates.
9. The average price of dairy products is expected to rise (late, significantly) in the next
year.
10. We are (currently, remarkably) unable to accept orders due to a technical.
C/ BASIC ADJECTIVE PHRASES AND ADVERBS PHRASES IN TOEIC
1. Be eligible for + Eligibility (n)
ST Be eligible to
do Eligible (a)
2. Be subject to +ST/ ing
3. Be familiar with (= be acquainted with)
Be accustomed
to (= be related to, be involved in )
4. Be associated with
5. Be responsible for
6. Be able to do ST
Ability (n)
7. Be compatible with
Compatibility (n) Compatibly (adv)
8. Late
Lately
9. Hard
Hardly
10. Right / promptly / immediately
before (after)
11. Approximately / about / almost
100 people
*** Vocabulary review.
1. The committee chose Sharon because she is (familiar, accustomed) with this field.
2. Requests for office supplies are subject to (approve, approval) from department heads.
3. By enlarging the dining area, we will be (able, capable) to provide better service.
4. Its stock price plummeted (immediately, already) after the merge announcement.
5. Only those who have worked more than a year will be (similar, eligible) for promotion.
6. The production team has been working (hard, hardy) to meet the deadline.
7. (About, seldom) 30 employees have shown their interest in the company soccer team.
8. Mr. Karl is (responsive, responsible) for translating official documents.
9. Consumer price are closely (involved, associated) with ordinary people‟s daily
consumption of ordinary goods.
10. Be sure to check if the software is (compatible, comparable) with our existing network
system.
11. Our new products are selling very well due to (favorite, favorable) conditions in the
market.
12. We have moved our office (late, lately) to a more convenient location.
13. His design was so (impressive, impressed) that the committee unanimously decided to
give Pierce the award.
PRACTICE TEST 02-----ADJECTIVES/ADVERBS
Chọn từ thích hợp và điền vào chỗ trống
1. Our special offer will be available to the first 100 customers because of
………..supplies.
A. Limited B. Speedy C. Available D. Persuasive

2. Be sure to read the directions…................before installing the software program.


A. Heavily B. Thoroughly C. Increasingly D. Readily

3. All employees are reminded to return….................merchandise to the manufacturer.


A. Lucrative B. Constructive C. Defective D. Effective

4. After three months of intensive negotiations, two companies…………agreed on the


terms of the merger contract.
A.Soon B. Finally C. Already D. Yet
5. Any Business directory is designed to provide a single…………..directory of local
businesses.
A. Transparent B. Apparent. C. Reliant D. Comprehensive

6. The training program on fire safety is scheduled to begin…................at 8:00 A.M and
end at 5:00P.M.
A. Promptly B. Carefully C. Openly D. Frequently

7. Please retain this note as a proof of ………..account ownership in case you open
another account in the future.
A. Future B. Open C. Prior D. New

8. The Ministry of Education has been training new teachers for the..............school year.
A. Once B. Current C. Upcoming D. First

9. According to a local newspaper, Digital Office. Com is planning to open a new store
………….located on Capital Avenue.
A. Conveniently B. Correctly C. Greatly D. Widely

10. The mileage you have accrued is…........for two years from the date of first use.
A. Fair B. Valid C. Neutral D. Level

11. We are considering switching suppliers because Vest Supplies has been…..late in
delivering our orders.
A. Steadily B. Officially C. Exactly D. Consistently

12. Because of the ……sales, the sales manager had to resign from his position.
A. Declining B. Rejected C. Potential D. Competitive

13. Professor Wang‟s achievements in Physics were remarkable,...................considering


that he didn‟t get any research funds for his project.
A. Favorably B. Unusually C. Especially D. Positively

14. Though one of the sizes you requested is currently..................., we will ship the rest of
your order immediately.
A. Related B. Stylish C. Disinterested D. Unavailable

15. Even though our new cellular phone model has been….............advertised, demand for
it is still relatively low.
A. Previously B. Sharply C. Adequately D. Widely

16. Because of the confidential nature of the contract, please keep it in a.....................place.
A. Cautions B. Secure C. Distinguished D. Strict

17. Though the digital camera you ordered is presently out of stock, we expect to receive
additional supplies…………..
A. Soon B. Nearly C. Almost D. Usually

18. It is very important for a negotiator to resolve disagreements by providing solutions


acceptable to the.........sides.
A. Affecting B. Criticizing C. Opposing D. Establishing

19. Because we take your privacy seriously, your private health records will be
kept…........confidential.
A. Completely B. Quickly C. Sincerely D. Regularly

20. The new exhibition is expected to feature the most………….selection of the ancient
artifacts.
A. Extensive B. Distracted C. Spacious D. Durable

Câu 21-23

mitted for the new marketing campaign. But, we are having difficulty making a decision among those proposals. I know you

company by helping

ase take time to review the 23proposals and provide your opinion about them.

23. A. Assembled Sincerely, B. Utilized C. Allowed D. Attacched


Jerry cooper.

Câu 24-26
Welcome
FTU offers undergraduate and graduate degree programs in Business Administration and
Management via distance learning.
Our degree programs are 24. …………designed for working professionals by leading
scholars from respected colleges and universities.
24. A. Tightly B. Randomly C. Slightly D. Carefully
We encourage you to browse through our web site to learn how you can benefit from our
self-paced, high-quality and 25......................degree programs.
25. A. Attentive B. Complimentary C. Affordable D. Foreseeable
Admission requirements: A Bachelor‟s degree in a 26. ………….field of study from an
accredited or state-approved college or university is required.
26. A. Dedicated B. Unlimited C. Detailed D. related
LESSON 07
A/ Phân biệt cơ bản giữa liên từ và giới từ
Liên từ Giới từ
- Chức năng chính: Liên kết - Chức năng chính: Liên kết
các mệnh đề trong câu các danh từ trong câu
 Because I was tired, I went home  Despite his efforts, he couldnt pass
early the exam
 The book on the table is boring
B/ Phân loại liên từ
PHẦN 1. LÝ THUYẾT
1, Liên từ sử dụng trong mệnh đề trạng ngữ
Liên từ Nghĩa
Thời gian Since Từ khi
When (=As) Khi
After Sau khi
Before Trước khi
As soon as Ngay sau khi
While Trong khi
Điều kiện If/ As long as Nếu/ Miễn là
Unless Trừ phi
Once Một khi
Nguyên nhân Because/ As/ Since Bởi vì
For Bởi vì
Now that Bởi vì
Nhƣợng bộ Although/ Even though/ Though/ Mặc dù/ Dù cho
Even if
Mục đích So/ So that Để mà
2, Liên từ kết hợp/ Liên từ kép
Liên từ kết hợp Liên từ kép
 And diễn tả sự kết hợp  Both A and B
 Or diễn tả sự lựa chọn  Neither A nor B
 But diễn tả sự tương phản  Either A or B
 Not only A but also B
C/ Các dạng bài tập liên từ
- Dạng 1: Câu hỏi tìm liên từ thích hợp: Với dạng bài tập này học viên cần
tìm ra đƣợc mối liên hệ giữa cụm từ đứng trƣớc và cụm từ đứng sau liên từ
để chọn liên từ phù hợp
 Our products are available at major department stores retail stores
 You already signed the contract you also need to sign the official
insurance documents
 Rents should be paid at before the end of each month.
- Dạng 2: Câu hỏi tìm từ thích hợp trƣớc và sau liên từ
a, Liên từ kết hợp các từ cùng loại
 The company plans to expand its operations and market share
b, Liên từ kết hợp các từ cùng hình thức
 The company plans to expand its operation and hire an experienced sales manager.
PHẦN 2. THỰC HÀNH
Chọn từ thích hợp điền vào chỗ trống
1. Ms. Lee will reserve the meeting room Ms. Leno prepares copies of the
meeting agenda
A. Also B. Than C. Moreover D. While
2. Successful applicants will be relocated to either New York London
A. Neither B. Nor C. Or D. Both
3. The new company building is relatively and adequately ventilated
A. Spacing B. Spaciousness C. Spaces D. Spacious
4. Due to your excellent performance, I will surely recommend your company to anyone
is planning a move
A. Who B. Which C. What D. When
5. Management consultants often stress workplace
rewarding dedicated employees‟
morale.
A. That B. Of C. In D. If
6. By sending gift certificates to regional managers, the director expressed his thanks for
their hard working _ commitment.
A. As B. And C. But D. Yet
7. the packages were sent by express delivery,the distributor has not yet
received them.
A. Although B. Unlike C. Already D. Meanwhile
8. The award was presented to Dr. Marson led the research on global warming.
A. While B. Who C. Which D. Whose
9. The city auditorium is available for private and business functions
A. Even B. Both C. Either D. Whether
10. The community center runs community-based programs include a wide
variety of classes and services
A. Whose B. Where C. That D. What
11. Please note that our service centers will be closed again at 3:00 pm on Friday
the upcoming holidays
A. Due to B. While C. Since D. As if
12. The sales director has suggested all sales reports be submitted by the end of
each week
A. On B. Even though. C. Which D. That
13. By volunteering, you can create lasting memories and valuable experience.
A. Gain B. Gaining C. Gained D. Gains
14. Trainees should contact their trainer directly they encounter any problem
beyond their control
A. If B. Which C. What D. Due to
15. the replacement parts arrive in the factory, we will restart the assembly
line.
A. Once B. Soon C. Then D. Later
16. Neither taking photographs bringing soft drinks is allowed in the museum.
A. Or B. Nor C. And D. Either
17. The budget report was due last Friday, the finance director has agreed to a short
extension.
A. But B. Beyond C. Until D. That
18. S & Accessories began to sell cosmetics products in 2002, now account for
more than 50 % of its total sales.
A. What B. Which C. If D. It
19. The company is having difficulty securing funds_ it has a lot of potential.
A. Which B. In addition to C.That D. Even though
Part VI: Chọn đáp án đúng cho câu 21-23 trong mẩu quảng cáo sau đây
Save 15 % with Student Advantage Discount Card
Passengers hold a Student Advantage Discount Card will immediately save 15 %
20. A. Which B. What C. That D. If
off rail fares to most of our destinations. Plus, cardholders can enjoy discounts on food,
clothing, travel, entertainment and more.
How to get a Student Advantage Discount Card?
Please visit the Student Advantage website to purchase a Student Advantage Discount
Card and more about the 15,000 discounts available around campus and online
21. A. Learn B. Learning C. Learns D. Learned
To receive this discount, the cardholder must present both a Student Advantage Discount
Card a school ID card. Each must be valid for the current school term.
22. A. But B. In fact C. Then D. And

Pacific Tribune Company


Pacific Tribune Company aims to provide and cultural experience that enrich our
community.
27. A. Educational B. Education C. Educate D. Educationally
We produce and support a range of public events are entertaining, informative and
most importantly, relevant to our readers and their lifestyles.
23. A. That B. They C. Once D. Who
Please contact us you want to know more about those events and join
them.
24. A. Who B. That C. And D. If.
LESSON 08
A/ COMMON CONJUNCTIONS IN TOEIC
1. if only if
If the management approves the budget, we will begin the construction.
2. once
Once we set up a task force, the team will investigate the problem.
3. before
Don‟t forget to turn off the lights before leaving the office.
4. since
Since it was damaged by the storm, the community center has been closed.
5. unless
Unless we take aggressive measures, the decline in sales will deteriorate.
6. while
While working part-time, Jason successfully finished his graduate studies.
7. although (= even if, even though, though)
Although he applied for several positions, Manny couldn‟t get a job.
8. because (= as, since)
We cannot accept credit payments because the card reader is out of order.
9. so that
The company opened a new in-house gym so that employees can use it at
any time.
10. as
Subcription rates for daily newspapers are expected to rise as paper prices
are on the rise.
*** Review
1. [Unless, Since] she joined our company, Ms. Kim has been working as the accounting
manager.
2. [Unit, If] your billing address has changed, please contact us as soon as possible.
3. This special offer is valid [after, while] supplies last.
4. Visitors cannot enter the laboratory [unless, although] they present proper identification.
5. [As, Once] we finalize the contract, we will set up a joint venture.
6. The proposal was rejected by the committee [because, so that] it was submitted too late.
7. We have to consider several factors [before, between] we make the final decision.
8. [As, While] it was discussed in the last meeting, construction of the new parking lot will
begin next week.
9. [Because, Although] she majored in history in the university, Kelly choose accountingas
her career.
10. The conference room is already reserved [so, since] we have to find another meeting
place.
B/ COMMON PREPOSITIONS IN TOEIC
1. by * by sales director
* by the end of the mouth
* (by V-ing): by comparing prices
* be caused by
2. for * for admission
* for ten years
* be responsible for
Demand for
Account for
Be known[noted] for
Reason for
Compete for
3. with * with a swimming pool
* with visitors
* Agree with ~comply with
4. concerning * (= regarding, about): concerning the advertisement
5. to * submit / return / send / lend A to B
6. within * within 24 hours
* within a radius of 10 miles
7. among * among the most trusted companies
* beetween A and B
Between the two companies
8. in * in the meeting room
* in three weeks
* experience in
* be interested in
Be involved in ~ invest in
9. due to * due to a mechanical problem
10. throughout * throughout the year
* throughout the country
11. through * through considerable efforts
* go through (= undergo)
12. on * on Tuesday
* have an effect [impact] on
* concentrate on ~ spend A on B
13. from * obtain / borrow A from B
* prohibit / prevent A from V-ing
*** Review
1. Please return the customer survey [within, by] 7 days of purchase.
2. The current high prices will have a negative impact [in, on] the economy.
3. The community center runs free English classes for residents [prior to, throughout] the
year.
4. Detailed information about the seminar will be provided [by, within] the sales manager.
5. There is a growing demand [for, through] experienced teachers.
6. The survey results will be released [of, in] two weeks.
7. Any questions [concerned, concerning] the revised tax law should be forwarded to the
accounting department.
8. S&P Co. ranks [among, at] the most influential consulting firms in the filed.
9. Please submit the required documents [by, to] our loan office.
10. The company announced thay will open a new hotel [with, for] brand-new fitness
center.
11. Travelers can obtain city guide books [form, to] one of the information centers.
12. You can be acquainted with the new system [on, through] frequent use.
13. The outdoor concert has been canceled [due to, concerning] the heavy rain.
C/ COMMON PREPOSITION PHRASES IN TOEIC
1. on behalf of
On behalf of the entire staff
2. in celebration of
In celebration of his promotion
3. at no cost (= for free)
Be provided at no cost
4. on one‟s way to
On his way to the conference center
5. in accordance with
in accordance with the company policy
6. in addition to
In addition to excellent benefits
7. according to
According to the article
8. under warranty
Be considered inder warranty
9. upon request
Be available up on request
10. by means of
By means of remote control
11. above one‟s
expectations Above the expert‟s expectations

12. in excess of In excess of 100 dollars


13. be noted for ~ (= be known for)
Be noted for excellent service
14. at the latest
By Friday at the lastest
15. at all times
Wear gloves at all times
*** Review

1. Construction workers should wear safety helmets [at, for] all times.
2. Product catalogs are available [at, upon] request.
3. Children under seven can enter the museum [at, with] no cost.
4. Tommy was transferred to Boston in accordance [at, with] his request.
5. Products wich are within a year from purchase are [about, under] warranty.
6. The second-quarter earnings were significantly [above, upon] the director‟s
expectations.
7. The president expressed her sincere thanks to all employees on behalf [of, to] the board
of directors.
8. We offer comprehensive benefits in addition [to, with] special incentives.
9. Maggie has been noted [for, through] her bold and creative designs.
10. [On, Of] their way to New York the delegates will stop over in the Tokyo office.
11. Please complete the budget report by this Wednesday [on, at] the latest.
12. A special luncheon will be held [in,at] celebration of the opening of the new plant.
13. All the heavy office equipment has been moved [in complicance with, by means of] a
crane.
14. Ms. Lee will be promoted [according to, regardless of] the company newsletter.
15. You need to report to customs officials if you have money in excess [over, of] $10,000.
D/ PRACTICE TEST 03
Part V: Complete the following sentences
1. There are a number of people who are interested………investing in overseas stocks
and real estate.
A. about B. on C. for D. In

2. Through our online graduate program, you can get your master‟s degree...............a
year.
A. instead of B. Within C. under D. Besides

3. Thanks to its strong performance, the company‟s stock price has nearly
doubled…............it went public twenty years ago.
A. if B. Since C. regarding D. Than

4. Please be aware that we are not able to process your loan application…........proper
documentation.
A. upon B. Until C. without D. along

5. Each month, one outstanding employee will be honored…………..his or her


exceptional contributions to our company.
A. for B. At C. across D. over

6. ……………..he finishes reviewing the accounting procedures, the auditor will go


over all the financial reports.
A. Whether B. Once C. As if D. yet

7. It has been decided by the joint committee to equally distribute the research
grant…...........the participating research institutions.
A. toward B. Among C. after D. during
8. This copier will bring you reliable and quality service.....................maintenance checks
are carried out periodically.
A. including B. Incase C. as long as D. unless

9. Employees are reminded to turn off their computer equipment….............leaving the


office.
A. before B. Until C.because D. then

10. It is advisable for employees to submit vacation preferences by next Friday.............the


latest.
A. at B. Before C. until D. up to

11. We have rescheduled the meeting………that the delegates who will arrive
tomorrow can attend.
A.so B. Concerning C. over D. since

12. Mr. Nelson has been given a challenging assignment, which is to lead the research
institute…............the process of restructuring.
A.behind B. Above C. except D. through

13. As you can see, I have included a list……………my previous supervisors in


addition to a cover letter and a résumé.
A. across B. To C. of D. among

14. Mr. Bright arrived at the conference center on time ……………his flight was
almost 30 minutes late.
A. as if B. Because C. ever though D. regardless of

15. The proposed youth center has a spacious recreational area........................a running
track.
A. across B. Beside C. with D. from

16.the new advertising campaign helps improve sales, the marketing team
will have to justify the expenses.
A. Unless B. Only C. whether D. while

17. Further question...................the merger announcement should be directly forwarded to


the public relations department.
A. relating B. Connecting C. concerning D. referring

18. For those experiencing difficulties with internet access, technical support is
available................the day.
A. from B. About C. between D. throughout
19. Creating a friendly working environment can help reduce personal stress…………..
increasing long – term productivity.
A. but B. Whether C. since D. while

20. We had to hire more than 30 sales personnel last month…………..we have noticed a
strong demand for our new products.
A. that B. If C. as D. which

Câu 21-23
SBA Business Customer Satisfaction Survey

Thank youvisiting this Website.


21. A. ThroughB. AsC. For D. Before
You have been randomly selected to take part in this surveythat is being conducted…Market research In
A. ByB. SinceC. WithoutD. To Market Research Inc. has been assigned to conduct the survey……………..
A. by means ofB. on behalf ofC. in observance ofD. in excess of. The U.S. Small business Association.
Please take a minute or two to provide us with your opinions. The feedback you provide
will help us serve you better in the future. All results are strictly confidential.
LESSON 09: ĐỌC HIỂU – EMAILS
A/ Câu hỏi tổng quan
1. Phân tích câu hỏi
Questions 1-2 refer to the following letter  Bài đọc này là 1 bức thư
Q1: What is the main purpose of this letter?  Câu hỏi mục đích. Mục đích thể
hiện ở phần đầu bức thư
Q2: Who sent this letter?  Câu hỏi về người viết thư. Thông tin này có thể tìm
thấy ở đầu or cuối bức thư
2. Phân tích bài đọc

ksonville Street
Thôn
g tin
về
ngƣời
that our address has changed due to the recent move to another office building.
nhận
decided to move our office which is now conveniently located on 5th Street. Please visit our website to find directions to ou
Mục
đích
của
bức
thƣ

Thông
tin về
ngƣời
gửi
thƣ
*** Thực hành phân tích lá thứ sau và trả lời các câu hỏi:
Q1. What is the main purpose of the e-mail?
Q2. To whom is this e-mail intended?
Q3. Who wrote the e-mail?

To: Paul Martins, Marketing Director of ANC Chemicals


Inc. From: Jeremy Heifers
Date: February 17
Subject: Marketing Seminar

Thank you for your invitation to the marketing seminar which will be held in
Royal Heritage Hotel.

I am very delighted to have been invited to the seminar as a guest speaker.


Please send me more specific information on the seminar with a detailed
schedule. Dont
hesitate to contact me if you have any specific request for the seminar address.

Regards,
Jeremy
Heifers
B/ Câu hỏi chi tiết và bổ sung
1. Phân tích câu hỏi
Questions 1-3 refer to the following email  Thư điện tử
Q1. What is attached to the email?
 Keywords: Attach, inlude, enclose
Q2. What are members asked to do?
 Keywords: Please, wan, ask, invite, require
Q3. Who is Dr. Parker?
 Keywords: Dr. Parker
2. Phân tích bài đọc

All members
m: Margaret Winston Date: July 20
ject: Regular meeting

is a reminder of the next meeting. We will hold the meeting at the Maxim Business center downtown.

Monday‟s meeting, Dr. Parker, who is one of the most successful venture investors in the field, will give a special lecture on
ase reply to this email if you wish to attend the lecture. For detailed information on Dr. Parker, refer to the attached profile.

garet Winston Thông tin về Dr. Parker

Nội dung yêu cầu

Thông tin về tài liệu đính kèm

*** Thực hành phân tích lá thƣ sau và trả lời câu hỏi phía dƣới
Any – Travel. Com
Mtb Networks Co.
158 Jefferson Road
Boston, MA 10914

Thank you for contacting Any-Travel.com. We have been in business for more than 10
years serving corporate clients.
As you requested, we have recommended several options for accommodations. Please
consult the enclosed documents regarding your travel plan. Once you choose one of
the hotels, we will begin confirming flight schedules and hotel reservations.
Please inform us of your decision by this Friday.
Sincerely,
Harry Cooper
Sales
1. What is enclosed with the letter?
Manager
2. What is Mr. Cooper asking MTB Networks to
do? C/ BÀI KIỂM TRA MÔ PHỎNG THỰC TẾ
Questions 1- 4 refer to the following e-mail.
To: Global Freight Inc.
From: Core Hosting
Co. Date: March 10
Subject: Domain Renewal

This is a reminder that your domain, www.globalfreight.com, will expire on March 30.
Please renew it before the expiration date, so you do not lose your exclusive right to the
domain name. for you convenience, our online renewal form is available at your
fingertips. Just click here to access the form.
There are three options to choose from one, two or five years. The charge is $50, $80 and
$200, respectively. We provide payment installment plans for business clients. If you
want to cancel your domain, please fill out and submit an online form. A charge of $20
will be added if you wish to change your domain.
Thank you for your
business. Henry Brook,
Core Hosting Co.

1. What is the purpose of the e-mail?


A. To promote an online renewal form
B. To inform a client of the expiration date
C. To sell a domain name
D. To encourage a company to change its domain
2. What will happen if the company doesn‟t renew the service?
A. It has to pay additional charges
B. Substantial fines will be assessed
C. Another firm might take over its name
D. It could lose its website
3. How much does a company pay if they want another address?
A. $20
B. $50
C. $80
D. $200
4. What must a company do to discontinue the service?
A. Fill out a form
B. Pay an additional charge
C. Sell its current domain
D. Download software
Questions 5- 7 refer to the following letter.
September 19
Mr. David Asher, President
Ford Construction Company
7083 Warren Sharon Rd
Brookfield, OH 44403
Dear Mr. Asher,
On behalf of the board of directors of Kelly Pharmaceutical Co., I would like to
express our sincere appreciation to Ford Construction Company for successfully
completing the reconstruction of our headquarters building.
Working under difficult conditions and tight schedules, your dedicated workers
finished the project on time, as you had promised.
I want to especially recognize your project manager, Fred Zook, who effectively led
your team of professional and skilled workers.
Sincerely,
Eugene Klassmeyer
CEO, Kelly Pharmaceutical Co.

5. What is the purpose of the letter?


A. To complain about poor construction
B. To celebrate a new company building
C. A announce a promotion
D. To express thanks for a fine job
6. What is mentioned as a problem of the construction?
A. Unfavorable weather conditions
B. A lack of skilled workers
C. Tight schedule
D. Financial constraint
7. Who is given special mention for leadership?
A. David Asher
B. Eugene Klaasmeyer
C. Fred Zook
D. A team of workers
Questions 8- 10 refer to the following e- mail
To: Larry
Thompson From:
Jason Harbor Date:
September 20
Subject: Room reservations

As you requested, I contacted several hotels to find a more comfortable place for
the annual shareholders‟ meeting scheduled on October 17. Among those
contacted, the Treasure Island Hotel does have the facilities that meet our needs.
I booked the Conference Room C which can easily accommodate 200 people; the
Abbey Room for press meetings, and the Rose Dining Room where the luncheon will be
served after the shareholders‟ meeting.
The following equipment and services are also needed:
- Three TV monitors
- A public address system
- Light refreshments
I will contact you next Monday to confirm the arrangements as outlined
above. Jason Harbor
Event Planning

8. What will be held after the shareholders‟ meeting?


A. A public address
B. A press conference
C. Lunch
D. Light refreshments
9. In which room will reporters most likely gather?
A. The president‟s office
B. Conference room C
C. Rose Dining Room
D. Abbey Room
10. Which of the following is NOT needed?
A. A projector
B. Drinks
C. Microphones and speakers
D. TV monitors
Questions 11- 15 refer to the following e-mail and notice
To: Kevin McBride
From: Taylor
Nelson Date: April
11
Subject: Your recent
order Dear Mr. McBride,
I‟m sorry to inform you that we are currently unable to accept your order because
your order dated April 7 used an expired price list.
In a notice issued March 10 (a copy of which is attached to this e-mail), we informed our
distributors of price increases on all products, which became effective April 1. Please
contact me as soon as possible to correct this problem.
We look forward to hearing from you
soon Sincerely,
Taylor Nelson,
Order
NOTICE
Due to current price increases in raw materials, J&C Electronics has decided to raise our
wholesale prices effective April 1. As of April 1, any order based on the current price
list connot be processed
Please contact the Oder Department for further questions. The price list will be
available on our website at www.jnc.com.

11. When was the notice issued?


A. March 10
B. April 1
C. April 7
D. April 11
12. What is attached to this e-mail?
A. An order form
B. A new price list
C. A notice
D. A current price list
13. In the email, what is Taylor Nelson asking Mr. McBride to do?
A. Visit the website
B. Use an online order form
C. Place a new order
D. Give him a call
14. In the notice, what is the reason for the company to raise prices?
A. Fierce competition
B. Low profitability
C. Expensive raw materials
D. New ordering system
15. What is NOT stated in the two passages?
A. The reason why the company cannot accept Mr. McBride‟s order
B. The date that the new prices became effective
C. The way to obtain the new price list
D. The items that Mr. McBride ordered
BÀI TẬP VỀ NHÀ: PART VII ETS 2016 TEST 1
LESSON 10: ĐỌC HIỂU – QUẢNG CÁO
A/ Câu hỏi tổng quan
1. Phân tích câu hỏi
Questions 1-2 refer to the following advertisement  Văn bản quảng cáo
Q1: What is this advertisement about?  Nội dung quảng cáo (Công việc, sản
phẩm, dịch vụ or bất động sản)
Q2: Who is most likely to be interested in the advertisement?  Đối tượng quảng
cáo này hướng tới
2. Phân tích bài đọc

cement depends more and more on your competency, not how long you‟ve worked. Employers expect employees to have
Quảng
cáo về
adults, many of whom already have extensive business experience. As a UW student, you will develop a personalized Acad
chương
trình học
thạc sĩ

Đối tượng
mà quảng
cáo
hướng tới

*** Thực hành phân tích và trả lời câu hỏi ở phía dƣới
Herald Software Inc. is a software company specializing in developing and
distributing educational software products. We are looking for a training manager for
our Training Department.
Job Description:
+ Developing management training curriculum
+ Organizing in-house staff development seminars
+ Coordinating external seminars in collaboration with outside
institutions Job Requirements:
+ A Bachelor‟s degree in education or related field
+ At least 3 years experience in adult training
+ Excellent communication skills
+ Good problem solving skills
We offer a competitive salary and comprehensive benefits package including tuition
reimbursement.
Please send your resume with two reference letters to recruiting@heraldsoft.com no later
than September 30. To find out more about the position, please visit our website at
www.heraldsoft.com.

1. What is the company‟s specialty?


2. What is stated as one of the job responsibilities?
3. What is NOT required for the position?
B/ Câu hỏi chi tiết và câu hỏi bổ sung
1. Phân tích câu hỏi
Questions 1-3 refer to the following advertisement  Bài đọc quảng cáo
Q1. What is NOT TRUE about the advertisement?  Câu hỏi phủ định, thông tin
không đúng về mẩu quảng cáo.
Q2. To take advantage of the offer, what is the minimum purchase requirement
for customers in Canada?  Câu hỏi chi tiết
Q3. To whom is the advertised offer not applied?  Câu hỏi phủ định, đối tượng
mà nội dung khuyến mãi ko nhắm tới.
2. Phân tích bài đọc
Put Off the Payments – Not the purchase Dành cho
khách thanh
toán bằng
No payments and no interest for 6 months on any purchase of thẻ tín dụng
$299 or more made on The Home Depot Consumer Credit
Card.
Lượng mua
Minimum purchase requirement of $299 or more in the US and tối thiểu cho
Puerto Rico, minimum purchase requirement of $450 in khách hàng ở
Canada
Canada.
Khu vực
không áp
Offer is not available in Mexico dụng
chương trình
khuyễn mại
*** Thực hành phân tích văn bản và trả lời câu hỏi phía dƣới
này
Aero Fitness Centers is one of the largest fitness companies in the industry.
Our exceptional people are the reason that we have shown consistent revenue growth
year after year. This commitment to excellence continues in our search for qualified sales
professionals!
SALES COUNSELOR
Job Descriptions:
A sales counselor is the key to our clients‟ success. The focus of the sales counselor is to
meet personal sales goals by assisting our clients to lose weight through one-on-one
counseling and the sale of our proven products, which include vitamins and nutritional
supplements.
Requirements:
Whar are the charateristics of a successful sales counselor?
*Persuasive * Energetic & Enthusiastic * Team Player * Customer Service Oriented
We prefer candidates who have previous sales experience or work experience in related
industries. Your energy and drive to succeed will be rewarded with a competitive
hourly wage and commission program. We also offer a comprehensive benefits
1. What is true about the company?
package.
2. What is stated as a reason for the company‟s success?
C/ BÀI KIỂM TRA MÔ PHỎNG THỰC TẾ
Questions 1- 3 refer to the following advertisement
Don‟t miss out on these great China Gateways- Call 1-877-738-2742
This fall, visit fascinating China. Stand in Tiananmen Square in the heart of Beijing to
watch the traditional raising of the flag to celebrate China‟s National Day on October 1.
Stay throughout the Golden Week and learn about China‟s history at popular sights such
as the Great Wall, and the Forbidden City.
7- Day Beijing Fall Special Gateway
$948- Departure Dates: 10/6, 10/7, 10/8
$998- Departure Dates: 9/26, 9/28, 10/3, 10/5

1. What is this advertisement about?


A. Airline tickets
B. Hotel reservations
C. Travel agencies
D. Travel products
2. Who is most likely to be interested in the advertisement?
A. College professors
B. Travelers who are planning to visit China
C. Students learning Chinese history
D. Residents in Beijing
3. Which of the following is NOT indicated in the advertisement?
A. Departure dates
B. Prices
C. Places to visit
D. Hotel names
Questions 4- 7 refer to the following job advertisement
Financial Assistant Needed
Prime Capital Co. is one of the fastest growing financial firms in the industry. We
are currently seeking financial assistants.
Duties:
- Assist with daily operations of financial advisors
- Review and analyze reports submitted from departments
- Prepare financial reports for financial
advisors Requirements:
- A college degree in Accounting or Finance
- Previous experience directly related to the duties outlined above
- Good computer skills
- MBA preferred
Applicants should send their resume and cover letter to recruiting@primecap.com
Please remember only those chosen for the interview will be contacted. No phone
calls. For further information about our company and the position, please visit our
website at www.primecap.com.

4. What kind of position is being advertised?


A. A financial advisor
B. A legal assistant
C. A senior manager
D. A financial assistant
5. What is NOT listed as a requirement for the position?
A. Working experience
B. A resume
C. A MBA degree
D. Computer abilities
6. How can people get more information about the company?
A. By calling the company
B. By visiting its website
C. By requesting information
D. By visiting the company
7. What is true about the recruiting process?
A. Applicants should send reference letters
B. Some applicants will have an interview
C. The company will conduct an interview over the phone
D. Application materials should be submitted in person
Questions 8- 10 refer to the following advertisement
Help center for tourists
Our goal is to provide tourists visiting our city with the following services.
*Information on the city including tourist attractions
*Free Interpretation
- English- speaking volunteers are on call 24 hours a day
*City Tour Bus
- The City Tour Bus runs a course that covers major points of interests in the city
- Buses come every 30 minutes
- It usually takes 2 hours to finish the tour.
To find more about the center, visit our website at www.infocenter.com

8. What is NOT a service provided by the help center?


A. Tour information
B. Tour bus
C. Free tour conductor
D. Interpretation
9. What can be inferred about the interpretation service?
A. It‟s available only in English
B. It‟s available only during weekdays
C. It‟s a paid service
D. It‟s operated by paid interpreters
10. What is NOT true about the tour bus?
A. It passes most tourist attractions in the city
B. It takes about 2 hours to finish
C. It comes every hour on the hour
D. It is operated by the help center

Questions 11- 15 refer to the following advertisement & review


Plasma TV
Overview:
HP delivers decades of technological leadership in a stylish high- definition TV.
Get 10% off any new HP television:
Add a new television to your online cart. After reviewing purchases in your shopping cart,
continue to the checkout page and insert coupon code TV6677. Coupon expires
1/30/2007.

11. What should a customer do to receive a discount?


A. Enter the coupon code
B. Present a coupon
C. Register as a new member
D. Recommend new customers
12. When does the offer end?
A. At the end of January
B. At the beginning of January
C. At the end of February
D. At the beginning of February
13. What is NOT stated as a reason for Danielle‟s satisfaction with the TV?
A. Affordable price
B. Easy of use
C. Discount offer
D. User- friendly
14. Why did Danielle buy the TV?
A. To replace an old one
B. To sell to another person
C. To donate to a local charity group
D. To give to someone as a gift

15. What can be implied about Danielle?


A. She gave the TV a poor rating
B. She read an article about the TV
C. She didn‟t receive the 10% discount
D. She is a regular customer of the store
BÀI TẬP VỀ NHÀ: PART VII ETS 2016 TEST 1
LESSON 11: THÔNG BÁO/ THƢ BÁO
A/ CÂU HỎI TỔNG QUAN
1. Phân tích câu hỏi
Questions 1-2 refer to the following memo  Văn bản thông báo nội bộ
Q1: What is the main purpose of the memo?  Mục đích của thông báo
Q2: Who issued the memo?  Người phát hành mẫu thông báo
2. Phân tích bài đọc

Thông tin về người gửi


ment Co. Date: November 27
Chủ để của thông báo
romotion of Mr. Hoffman to the position of senior marketing manager in our international marketing division.

ffective international market- ing strategies and overseeing regional offices throughout Asia. Mục đích của thông báo
years ago and was promoted to marketing manager last year. He was transfered to the branch office in Hong Kong last ye

*** Thực hành phân tích đoạn văn và trả lời câu hỏi ở dưới
WARRANTY INFORMATION
The warranty on our chairs ranges from one year to lifetime warranties. Most
warranties cover full replacement for at least three years. Specific warranties are stated
on the sales receipt.
To find out if you qualify for our warranty service, please call our customer service
center or visit our website. Please have your warranty card, product model, and receipt
1. What is this notice about?
ready.
2. Who is this notice most likely aimed at?
3. What should people prepare when calling the service center?
B/ Câu hỏi chi tiết và câu hỏi bổ sung
1. Phân tích câu hỏi
Questions 1-2 refer to the following notice?  Văn bản thông báo
Q1. Which customers cannot use the delivery service?  Đây là câu hỏi phủ định. Đáp
án là đối tượng khách hàng không được hưởng dịch vụ giao hàng
Q2. What is true about the notice?  Đây là câu hỏi chi tiết.
2. Phân tích bài đọc
FREE APPLIANCE DELIVERY
Offer is valid until March 29, 2007 by mail-in rebate with any Tiêu chuẩn để
được hưởng
dịch vụ giao
hàng
e purchase totaling $300 or more. Normal delivery terms apply.

is not available in some areas, like Alaska and Hawaii. Đối tượng khách hàng không được hưởng dịch vụ giao h

valid on purchases from Appliance Direct stores and appliancedirect.com only. See rebate form for redemption details. Max

Số tiền tối đa được hoàn lại

*** Thực hành phân tích và trả lời câu hỏi phía dưới
MEMORANDUM
To: All Employees
From: Elliot Morgan, Human Resources Director
Date: February 19
Subject: Vacant Position
Because of the Ms. Loren‟s sudden resignation, the Network Administration position is
currently vacant. According to our company‟s in-house recruiting policy, qualified
employees are encouraged to apply for the position in the Information Technology
Department.
If interested, please submit the following documents to the Human Resources
Department no later than March 5.
 An application form available from the Human Resources office
 A reference letter from your current supervisor
 A resume
The successful candidate will be notified by March 15. For a more detailed job
description, please visit our office.
Elliot Morgan
1. What is NOT listed as required documents?
2. When is the deadline for submitting the application?
3. To get more information about the position, what should employees do?
C/ BÀI KIỂM TRA MÔ PHỎNG THỰC TẾ
Questions 1- 3 refer to the following notice
Return Policy
It is a must to contact us before returning the item(s). Items returned without our
prior approval will not be processed.
30 days Money Back policy
Shipping, handling fee and insurance are NOT refundable in any way. Item must be
returned to us within 30 days of your purchase. Returns are refunded through money
order ONLY. If you choose other payment method, sorry, we CANNOT refund the item.
30 days Warranty
All defective items will be replaced with brand new items for the first 30 days of your
purchase. Buyer is responsible for the return shipping charge ONLY if the defect is
the factory‟s fault or damaged during shipping. New item will be sent within 5- 7
business days after receiving your returned item.
1. What should customers do before asking for a refund?
A. They should return the item
B. They should contact the company
C. They should pay shipping charges
D. They should submit a receipt
2. What is NOT true about the 30 days Money Back Policy?
A. Insurance will not be refunded
B. Items must be returned within 30 days
C. Refunds will be deposited into an account
D. Handling fees are not refundable
3. In which case is an item exchanged for a brand new item?
A. Items unopened
B. Items under warranty
C. Items with defects
D. Items with a receipt
Questions 4- 6 refer to the following
memo
MEMO
To: All employees
From: Tylor Johns
Date: May 20
Subject: Parking
lot
This is an announcement that the management has approved a plan to repave the employee
parking lot. The work will begin on June 1. During the work, employees are not allowed
to use the parking lot. We strongly recommend taking public transportation. Employees
can use a parking lot in the Commerce Building for $ 100 a month. The work should be
done by June 15.
Please accept our apologies for the inconvenience and thank for your
cooperation. Tylor Johns
Facilities Manager
4. What is the purpose of the memo?
A. To announce the opening of the new employee parking lot.
B. To inform staff of maintenance work
C. To thank employees for their cooperation
D. To announce a partnership
5. How long will the construction take?
A. Several days
B. One week
C. Two weeks
D. A month
6. What inconvenience will employees face in June?
A. They should work overtime
B. They are prohibited from using the parking lot
C. The parking rates will be raised to $100
D. They should collaborate in the repaving
work Questions 7- 9 refer to the following notice
Library Fines and Fees
Overdue Materials:
- Books, audio books, Music CDs, Magazines:
 $0.10 per day per item
 $ 5.00 maximum fine per item
- Video Cassettes & DVDs:
 $1.00 per day per day per item
 $5.00 maximum fine per
item Lost or Damaged Materials:
- Replacement cost plus $5.00 processing fee
- For items whose cost is undeterminable, the default replacement cost is $25.00 plus
$5.00 processing fee
- An exact replacement for a lost item will be accepted, but a $5.00 processing
fee will still be assessed.

7. How much does a customer have to pay for overdue magazines?


A. $0.10 per day per item
B. $1 per day per item
C. $5 per day per item
D. $25
8. What is true about lost or damaged materials?
A. Maximum fine is $5
B. A processing fee is charged in any case.
C. Replacement cost is $5
D. An exact replacement is possible at no cost
9. How much will be charged for lost materials whose original prices are
not available?
A. $1
B. $5
C. $25
D. 30
Questions 10- 14 refer to the following notice and e-mail
NOTICE
A seminar on Current Social & Political Issues will be held;
Presenter Topic
John McGuire Market values and social values
Stephen Reverse The responsibility of intellectuals
Dennis Florig Power and Terror; A videotaped
interview

Date: June 3, 2:00- 5:00 P.M


Place: Seminar room 203, Louis University

For more information, please send an e-mail to the e-mail address below. Your ideas and
questions are always welcome.

The society for Social & Political Studies


E-mail: aaronsmith@ssps.org

To:
aaronsmith@ssps.org
From: Morgan Hoffman
Date: May 28
Subject: Seminar on Current Social & Political Issues

Hi, I‟m Morgan Hoffman majoring in Social and Political Economics in Louis University.
Professor McKinsey recommended I attend the seminar you are organizing. I‟m really
interested in attending the seminar on current social and political issues. I wonder if there
is anything I have to bring. Please let me know.

Morgan Hoffman
10. Who is going to speak about the responsibility of the intellectuals?
A. Aaron Smith
B. John McGuire
C. Stephen Reverse
D. Dennis Florig
11. When will a visual aid be used?
A. John McGuire‟s lecture
B. Dennis Florig‟s lecture
C. During the Q & A session
D. After the seminar
12. How can someone attend the seminar?
A. By registering at the seminar
B. By suggesting ideas
C. By sending an e-mail
D. By participating in the seminar
13. What does Mr. Hoffman want to know about?
A. How to contact the speakers
B. Where to go
C. What to prepare
D. How to attend
14. How did Mr. Hoffman learn about the seminar?
A. From a faculty member
B. From the notice
C. From his friend
D. From the organizer
BÀI TẬP VỀ NHÀ: PART VII ETS 2016 TEST 1

You might also like